OB test bank questions TEST 3

Pataasin ang iyong marka sa homework at exams ngayon gamit ang Quizwiz!

Chapters 13, 14, and 15

See the questions up next :)

13) A pregnant couple have been notified that their 32-week fetus is dead. The father is yelling at the staff, and his wife is crying uncontrollably. Their 5-year-old daughter is banging the head of her doll on the floor. Which nursing action would be most helpful at this time? 1. Tell the father that his behavior is inappropriate. 2. Sit with the family and quietly communicate sorrow at their loss. 3. Help the couple to understand that their daughter is acting inappropriately. 4. Encourage the couple to send their daughter to her grandparents.

Answer 2 Explanation: 2. Sitting down for a moment with the woman and her partner and acknowledging the loss in the event of a known demise or impending death will go a long way toward establishing a relationship of trust between the nurse and the parents.

6) It is 1 week before a pregnant clients due date. The nurse notes on the chart that the clients pulse rate was 74-80 before pregnancy. Today, the clients pulse rate at rest is 90. What action should the nurse should take? 1. Chart the findings. 2. Notify the physician of tachycardia. 3. Prepare the client for an electrocardiogram (EKG). 4. Prepare the client for transport to the hospital.

Answer: 1 Explanation: 1. The pulse rate frequently increases during pregnancy, although the amount varies from almost no increase to an increase of 10 to 15 beats per minute. This is a normal response, and does not indicate a need for emergency measures or treatment.

19) The nurse notes the following findings in a client at 12-weeks gestation. Which of the findings would enable the nurse to tell the client that she is diagnostically pregnant? 1. Fetal heart rate by Doppler 2. Positive pregnancy test 3. Positive Chadwicks sign 4. Montgomery gland enlargement

Answer: 1 Explanation: 1. A fetal heart rate by Doppler is a diagnostic (positive) change of pregnancy.

4) A client and her husband have contacted their physician about fertility problems. At the initial visit, the nurse instructs them about the infertility workup. Which statement by the client would indicate that the instructions have been successful? 1. The first test that we need to schedule is a semen analysis. 2. We need to schedule the Pap smear test first. 3. We need to schedule an appointment with the social worker in order to adopt. 4. We need to schedule an appointment with a marriage counselor.

Answer: 1 Explanation: 1. A semen analysis is one of the first diagnostic tests, prior to doing invasive procedures.

18) A client tells you that her mother was a twin, two of her sisters have twins, and several cousins either are twins or gave birth to twins. The client, too, is expecting twins. Because there is a genetic predisposition to twins in her family, there is a good chance that the client will have what type of twins? 1. Dizygotic twins 2. Monozygotic twins 3. Identical twins 4. Nonzygotic twins

Answer: 1 Explanation: 1. Studies indicate that dizygotic twins tend to occur in certain families, perhaps because of genetic factors that result in elevated serum gonadotropin levels leading to double ovulation.

31) The nurse asks a woman how her husband is dealing with the pregnancy. The nurse concludes that counseling is needed when the woman makes which statement? 1. My husband is ready for the pregnancy to end so that we can have sex again. 2. My husband is much more attentive to me now that I am pregnant. 3. My husband seems more worried about our finances now than he was before the pregnancy. 4. My husband plays his favorite music for my belly so the baby will learn to like it.

Answer: 1 Explanation: 1. This is implying that the woman and her husband are not having sex, which indicates the need for counseling. Sex is fine with a normal pregnancy.

9) The prenatal clinic nurse is explaining test results to a client who has had an assessment for fetal well-being. Which statement indicates that the client understands the test result? 1. The normal Doppler velocimetry wave result indicates my placenta is getting enough blood to the baby. 2. The reactive non-stress test means that my baby is not growing because of a lack of oxygen. 3. Because my contraction stress test was positive, we know that my baby will tolerate labor well. 4. My biophysical profile score of 6 points to everything being normal and healthy for my baby.

Answer: 1 Explanation: 1. A decrease in fetal cardiac output or an increase in resistance of placental vessels will reduce umbilical artery blood flow. Doppler velocimetry is best used when intrauterine growth restriction is diagnosed; therefore, the baby is getting an adequate blood supply.

18) A 27-year-old married woman is 16 weeks pregnant and has an abnormally low maternal serum alpha-fetoprotein test. Which statement indicates that the couple understands the implications of this test result? 1. We have decided to have an abortion if this baby has Down syndrome. 2. If we hadnt had this test, we wouldnt have to worry about this baby. 3. Ill eat plenty of dark green leafy vegetables until I have the ultrasound. 4. The ultrasound should be normal because Im under the age of 35.

Answer: 1 Explanation: 1. A low maternal serum alpha-fetoprotein test can indicate trisomy 18 or trisomy 21 (Down syndrome). Many couples abort a fetus that has a genetic abnormality that significantly affects quality of life or has multiple medical problems. Down syndrome is more likely to occur in the fetuses of women over the age of 35 at delivery, but is not limited to this age group.

13) A primary herpes simplex infection in the first trimester can increase the risk of which of the following? 1. Spontaneous abortion 2. Preterm labor 3. Intrauterine growth restriction 4. Neonatal infection

Answer: 1 Explanation: 1. A primary herpes simplex infection can increase the risk of spontaneous abortion when infection occurs in the first trimester.

18) The nurse is planning an educational session for pregnant vegans. What information should the nurse include? 1. Eating beans and rice provides complete protein needs. 2. Soy is not a good source of protein for vegans. 3. Rice contains a high level of vitamin B12. 4. Vegan diets are excessively high in iron.

Answer: 1 Explanation: 1. Adequate dietary protein can be obtained by consuming a varied diet with adequate caloric intake and plant-based proteins. Consuming an assortment of plant proteins throughout the day such as beans and rice, peanut butter on whole-grain bread, and whole-grain cereal with soy milk ensures that the expectant mother obtains all essential amino acids.

19) The nurse is preparing a client for amniocentesis. Which statement would indicate that the client clearly understands the risks of an amniocentesis? 1. I might go into labor early. 2. It could produce a congenital defect in my baby. 3. Actually, there are no real risks to this procedure. 4. The test could stunt my babys growth.

Answer: 1 Explanation: 1. Amniocentesis has the potential for causing a spontaneous abortion.

22) The pregnant client and her partner are both 40 years old. The nurse is explaining the options of chorionic villus sampling (CVS) and amniocentesis for genetic testing. The nurse should correct the client if she makes which statement? 1. Amniocentesis results are available sooner than CVS results are. 2. CVS carries a higher risk of limb abnormalities. 3. Amniocentesis cannot detect a neural tube defect. 4. CVS is performed through my belly or my cervix.

Answer: 1 Explanation: 1. Amniocentesis results take longer to process than do CVS results.

15) A pregnant client who was of normal prepregnancy weight is now 30 weeks pregnant. She asks the nurse what appropriate weight gain for her should be. What is the nurses best response? 1. 25-35 pounds 2. 30-40 pounds 3. 17-18 pounds 4. Less than 15 pounds

Answer: 1 Explanation: 1. An appropriate weight gain for a woman of normal weight before pregnancy would be 25-35 pounds.

15) The nurse has completed a community presentation about the changes of pregnancy, and knows that the lesson was successful when a community member states that which of the following is one probable or objective change of pregnancy? 1. Enlargement of the uterus 2. Hearing the babys heart rate 3. Increased urinary frequency 4. Nausea and vomiting

Answer: 1 Explanation: 1. An examiner can perceive the objective (probable) changes that occur in pregnancy. Enlargement of the uterus is a probable change.

25) Which statement is best to include when teaching a pregnant adolescent about nutritional needs of pregnancy? 1. It is important to eat iron-rich foods like meat every day. 2. Calcium and milk arent needed until the third trimester. 3. Folic acid intake is the key to having a healthy baby. 4. You just need to pay attention to what you eat now.

Answer: 1 Explanation: 1. An inadequate iron intake is a major concern with the adolescent diet. Iron needs are high for the pregnant teen because of the requirement for iron by the enlarging maternal muscle mass and blood volume. Giving specific examples is helpful when giving nutritional information.

15) The postpartum unit nurse is caring for a client who delivered a term stillborn infant yesterday. The mother is heard screaming at the nutrition services worker, This food is horrible! You people are incompetent and cant cook a simple edible meal! The nurse understands this as which of the following? 1. An indication the mother is in the anger phase of grief. 2. An abnormal response to the loss of the child. 3. Reactive stress management techniques in use. 4. Denial of the death of the child she delivered yesterday.

Answer: 1 Explanation: 1. Anger, resulting from feelings of loss, loneliness, and, perhaps, guilt, is a common reaction. Anger may be projected at significant others and/or healthcare team members. Page Ref: 946, 947

6) A couple is requesting fertility counseling. The nurse practitioner has identified the factors listed below in the womans health history, and knows which of them could be contributing to the couples infertility? 1. The client is 38 years old. 2. The client was 13 years old when she started her menses. 3. The client works as a dental hygienist 3 days a week. 4. The client jogs 2 miles a day.

Answer: 1 Explanation: 1. As the eggs of older women age, their fertility is reduced.

28) A woman has a hydatidiform mole (molar pregnancy) evacuated, and is prepared for discharge. The nurse should make certain that the client understands that what is essential? 1. That she not become pregnant until after the follow-up program is completed 2. That she receive RhoGAM with her next pregnancy and birth 3. That she has her blood pressure checked weekly for the next 30 days 4. That she seek genetic counseling with her partner before the next pregnancy

Answer: 1 Explanation: 1. Because of the risk of choriocarcinoma, the woman treated for hydatidiform mole should receive extensive follow-up therapy. Follow-up care includes a baseline chest X-ray to detect lung metastasis and a physical examination including a pelvic examination. The woman should avoid pregnancy during this time because the elevated hCG levels associated with pregnancy would cause confusion as to whether cancer had developed.

The nurse is explaining to a new prenatal client that the certified nurse-midwife will perform clinical pelvimetry as a part of the pelvic exam. The nurse knows that teaching has been successful when the client makes which statement about the reason for the exam? 1. It will help us know how big a baby I can deliver vaginally. 2. Doing this exam is a part of prenatal care at this clinic. 3. My sister had both of her babies by cesarean. 4. I am pregnant with my first child.

Answer: 1 Explanation: 1. By performing a series of assessments and measurements, the examiner assesses the pelvis vaginally to determine whether the size and shape are adequate for a vaginal birth; this procedure is called clinical pelvimetry.

26) At her first prenatal visit, a woman is discussing fetal development with the nurse. The client asks, When will my baby actually have a heartbeat? The nurse should say the heartbeat of an embryo is distinguishable by what time? 1. The fourth week 2. The sixth week 3. The eighth week 4. The twelfth week

Answer: 1 Explanation: 1. By the end of the fourth week, embryonic blood is circulating between the embryo and the chorionic villi.

13) What type of testing is an inexpensive way to predict the presence of tubal disease and may be more predictive of infertility than an abnormal HSG? 1. Chlamydia trachomatis IgG antibody testing 2. Preimplantation genetic testing 3. Noninvasive prenatal testing (NIPT) 4. DNA testing

Answer: 1 Explanation: 1. Chlamydia trachomatis IgG antibody testing is an inexpensive way to predict the presence of tubal disease and may be more predictive of infertility than an abnormal HSG.

31) During a prenatal exam, a client describes several psychosomatic symptoms and has several vague complaints. What could these behaviors indicate? 1. Abuse 2. Mental illness 3. Depression 4. Nothing, they are normal

Answer: 1 Explanation: 1. Chronic psychosomatic symptoms and vague complaints can be indicators of abuse.

A 43-year-old client has just had a positive pregnancy test. She cries, and states, I just dont know what Ill do. I cant be pregnant. Which nursing diagnosis would be the most appropriate? 1. Decisional Conflict related to unexpected pregnancy 2. Knowledge, Deficient related to advanced maternal age 3. Depression related to unexpected pregnancy 4. Health Maintenance, Ineffective related to advanced maternal age

Answer: 1 Explanation: 1. Decisional Conflict related to unexpected pregnancy is the most appropriate nursing diagnosis.

14) The client at 37 weeks gestation calls the clinic nurse to report that neither she nor her partner has felt fetal movement for the past 48 hours. The nurse anticipates that the physician will order which test to assess fetal viability? 1. Ultrasound 2. Serum progesterone levels 3. Computed tomography (CT) scan 4. Contraction stress test

Answer: 1 Explanation: 1. Diagnosis of intrauterine fetal death (IUFD) is confirmed by visualization of the fetal heart with absence of heart action on ultrasound. pg 944

5) The nurse is presenting a preconception counseling class. The nurse instructs the participants that niacin intake should increase during pregnancy to promote metabolic coenzyme activity. The nurse will know that teaching has been effective if a client suggests which food as a source of niacin? 1. Fish 2. Apples 3. Broccoli 4. Milk

Answer: 1 Explanation: 1. Dietary sources of niacin include meats, fish, and whole grains.

19) A client has delivered a stillborn child at 26 weeks gestation. She tells the nurse that none of her friends have called or visited, and that her husbands parents seem unwilling to talk about the loss. The nurse recognizes the mothers grief as which of the following? 1. Disenfranchised grief 2. Bereavement 3. An intuitive style of coping 4. Denial

Answer: 1 Explanation: 1. Disenfranchised grief is not supported by the usual societal customs. People are uncomfortable discussing the loss with the parents and often pull away when their support is most needed. Page Ref: 945

15) Doppler flow studies (umbilical velocimetry) help to assess which of the following? 1. Placental function and sufficiency 2. Fetal heart rate 3. Fetal growth and fluid levels 4. Maturity of the fetal lungs

Answer: 1 Explanation: 1. Doppler flow studies (umbilical velocimetry) help to assess placental function and sufficiency. Uteroplacental insufficiency is a risk for a woman with preeclampsia. If fetal growth restriction is present, Doppler velocimetry of the umbilical artery is useful for fetal surveillance.

23) The nurse is working with a client who has experienced a fetal death in utero at 20 weeks. The client asks what her baby will look like when it is delivered. Which statement by the nurse is best? 1. Your baby will be covered in fine hair called lanugo. 2. Your child will have arm and leg buds, not fully formed limbs. 3. A white, cheesy substance called vernix caseosa will be on the skin. 4. The genitals of the baby will be ambiguous.

Answer: 1 Explanation: 1. Downy fine hair called lanugo covers the body of a 20-week fetus.

14) A woman is being treated for preterm labor with magnesium sulfate. The nurse is concerned that the client is experiencing early drug toxicity. What assessment finding by the nurse indicates early magnesium sulfate toxicity? 1. Patellar reflexes weak or absent 2. Increased appetite 3. Respiratory rate of 16 4. Fetal heart rate of 120

Answer: 1 Explanation: 1. Early signs of magnesium sulfate toxicity are related to a decrease in deep tendon reflexes.

The nurse is working with male teens whose partners are pregnant. What statement by the father-to-be requires that the nurse intervene? 1. He will be the only other person who will be present for the birth, although his girlfriend wants her mother to be with her. 2. He was very sexually active at an earlier age and he has had more sexual partners than his girlfriend. 3. The pregnancy does not seem real to him, and he is not sure what he should do to plan for the future. 4. He does not want to be married.

Answer: 1 Explanation: 1. Even if the adolescent father has been included in the health care of the client throughout the pregnancy, it is not unusual for her to want her mother as her primary support person during labor and birth. Overriding his girlfriends expressed desire could be an indication that their relationship is abusive.

A 38-year-old client in her second trimester states a desire to begin an exercise program to decrease her fatigue. What is the most appropriate nursing response? 1. Fatigue should resolve in the second trimester, but walking daily might help. 2. Avoid a strenuous exercise regimen at your age. Drink coffee to combat fatigue. 3. Avoid an exercise regimen due to your pregnancy. Try to nap daily. 4. Fatigue will increase as pregnancy progresses, but running daily might help.

Answer: 1 Explanation: 1. Even mild to moderate exercise is beneficial during pregnancy. Regular exerciseat least 30 minutes of moderate exercise daily or at least most days of the weekis preferred.

26) The nurse is supervising care by a new graduate nurse who is working with a couple who have experienced a stillbirth. Which statement made by the new nurse indicates that further instruction is necessary? 1. I should stay out of their room as much as possible. 2. The parents might express their grief differently from each other. 3. My role is to help the family communicate and cope. 4. Hopelessness might be expressed by this family.

Answer: 1 Explanation: 1. Families experiencing perinatal loss need support. The nurse should stay with the couple so they do not feel alone and isolated; however, cues that the couple wants to be alone should be assessed continuously.

17) The prenatal clinic nurse is caring for a 15-year-old client who is at 8 weeks gestation. The client asks the nurse why she is supposed to gain so much weight. What is the best response by the nurse? 1. Gaining 25-35 pounds is recommended for healthy fetal growth. 2. Its what your certified nurse-midwife recommended for you. 3. Inadequate weight gain delays lactation after delivery. 4. Weight gain is important to ensure that you get enough vitamins.

Answer: 1 Explanation: 1. For an appropriate-weight woman, 25-35 pounds of weight gain is recommended for optimal fetal growth and development.

The 19-year-old pregnant woman begins a job to save money for the baby. What is the most significant developmental task the nurse understands this statement to demonstrate? 1. Striving for gaining autonomy and independence 2. Completed development of a sense of identity 3. Attainment of a sense of achievement 4. Having developed an intimate relationship

Answer: 1 Explanation: 1. Having a job is how most teens develop financial independence and autonomy.

The nurse at the prenatal clinic has four calls to return. Which phone call should the nurse return first? 1. Client at 32 weeks, reports headache and blurred vision. 2. Client at 18 weeks, reports no fetal movement in this pregnancy. 3. Client at 16 weeks, reports increased urinary frequency. 4. Client at 40 weeks, reports sudden gush of fluid and contractions.

Answer: 1 Explanation: 1. Headache and blurred vision are signs of preeclampsia, which is potentially life-threatening for both mother and fetus. This client has top priority.

33) A nurse is discussing diet with a pregnant woman. Which food should the nurse advise the client to avoid during her pregnancy? 1. Bologna 2. Cantaloupe 3. Spinach 4. Cornbread

Answer: 1 Explanation: 1. Hot dogs and other luncheon meats should not be eaten during pregnancy unless they are fully cooked.

) The nurse is caring for a pregnant woman who admits to using cocaine and ecstasy on a regular basis. The client states, Everybody knows that alcohol is bad during pregnancy, but whats the big deal about ecstasy? What is the nurses best response? 1. Ecstasy can cause a high fever in you and therefore cause the baby harm. 2. Ecstasy leads to deficiencies of thiamine and folic acid, which help the baby develop. 3. Ecstasy produces babies with small heads and short bodies with brain function alterations. 4. Ecstasy produces intrauterine growth restriction and meconium aspiration.

Answer: 1 Explanation: 1. Hyperthermia (elevated temperature) is a side effect of MDMA (ecstasy).

3) A womans history and appearance suggest drug abuse. What is the nurses best approach? 1. Ask the woman directly, Do you use any street drugs? 2. Ask the woman whether she would like to talk to a counselor. 3. Ask some questions about over-the-counter medications and avoid mention of illicit drugs. 4. Explain how harmful drugs can be for her baby.

Answer: 1 Explanation: 1. If drug abuse is suspected, the nurse should ask direct questions and be matter-of-fact and nonjudgmental to elicit honest responses.

27) A 28-year-old woman has been an insulin-dependent diabetic for 10 years. At 36 weeks gestation, she has an amniocentesis. A lecithin/sphingomyelin (L/S) ratio test is performed on the sample of her amniotic fluid. Because she is a diabetic, what would an obtained 2:1 ratio indicate for the fetus? 1. The fetus may or may not have immature lungs. 2. The amniotic fluid is contaminated. 3. The fetus has a neural tube defect. 4. There is blood in the amniotic fluid.

Answer: 1 Explanation: 1. Infants of diabetic mothers (IDMs) have a high incidence of false-positive results (i.e., the L/S ratio is thought to indicate lung maturity, but after birth the baby develops RDS).

18) Infants of women with preeclampsia during pregnancy tend to be small for gestational age (SGA) because of which condition? 1. Intrauterine growth restriction 2. Oliguria 3. Proteinuria 4. Hypertension

Answer: 1 Explanation: 1. Infants of women with preeclampsia during pregnancy tend to be small for gestational age (SGA) because of intrauterine growth restriction. The cause is related specifically to maternal vasospasm and hypovolemia, which result in fetal hypoxia and malnutrition.

8) The client with insulin-dependent type 2 diabetes and an HbA1c of 5.0% is planning to become pregnant soon. What anticipatory guidance should the nurse provide this client? 1. Insulin needs decrease in the first trimester and usually begin to rise late in the first trimester as glucose use and glycogen storage by the woman and fetus increase. 2. The risk of ketoacidosis decreases during the length of the pregnancy. 3. Vascular disease that accompanies diabetes slows progression. 4. The baby is likely to have a congenital abnormality because of the diabetes.

Answer: 1 Explanation: 1. Insulin needs decrease in the first trimester and usually begin to rise late in the first trimester as glucose use and glycogen storage by the woman and fetus increase.

15) The nurse is planning to teach couples factors that influence fertility. Which factor should not be included in the teaching plan? 1. Sexual intercourse should occur every day of the week. 2. Get up to urinate 1 hour after intercourse. 3. Do not douche. 4. Institute stress-reduction techniques.

Answer: 1 Explanation: 1. It is optimal if sexual intercourse occurs every other day during the fertile period.

9) In evaluating information taught about conception and fetal development, the client verbalizes understanding about transportation time of the zygote through the fallopian tube and into the cavity of the uterus with which statement? 1. It will take at least 3 days for the egg to reach the uterus. 2. It will take 8 days for the egg to reach the uterus. 3. It will only take 12 hours for the egg to go through the fallopian tube. 4. It will take 18 hours for the fertilized egg to implant in the uterus.

Answer: 1 Explanation: 1. It will take at least 3 days for the egg to reach the uterus is the correct statement.

12) The nurse is responding to phone calls. Whose call should the nurse return first? 1. A client at 37 weeks gestation reports no fetal movement for 24 hours. 2. A client at 29 weeks gestation reports increased fetal movement. 3. A client at 32 weeks gestation reports decreased fetal movement X 2 days. 4. A client at 35 weeks gestation reports decreased fetal movement X 4 hours.

Answer: 1 Explanation: 1. Lack of fetal movement can be an indication of nonreassuring fetal status or even fetal death. This client is the highest priority.

28) The nurse is preparing a prenatal class about infant feeding methods. The maternal nutritional requirements for breastfeeding and formula-feeding will be discussed. What statement should the nurse include? 1. Breastfeeding requires a continued high intake of protein and calcium. 2. Formula-feeding mothers should protect their health with a lot of calcium. 3. Producing breast milk requires calories, but any source of food is fine. 4. Formula-feeding mothers need a high protein intake to avoid fatigue.

Answer: 1 Explanation: 1. Lactation requires calories, along with increased protein and calcium intake.

The primiparous client has told the nurse that she is afraid she will develop hemorrhoids during pregnancy because her mother did. Which statement would be best for the nurse to make? 1. It is not unusual for women to develop hemorrhoids during pregnancy. 2. Most women dont have any problem until after theyve delivered. 3. If your mother had hemorrhoids, you will get them, too. 4. If you get hemorrhoids, you probably will need surgery to get rid of them.

Answer: 1 Explanation: 1. Many pregnant women will develop hemorrhoids. Hemorrhoids are varicosities of the veins in the lower end of the rectum and anus. During pregnancy, the gravid uterus presses on the veins and interferes with venous circulation. As the pregnancy progresses, the straining that accompanies constipation can contribute to the development of hemorrhoids.

The nurse is working with a group of pregnant teens. Which statement indicates that teaching has been successful? 1. Pregnant teens are more likely to quit school prior to graduation. 2. Because I am young, I have a low risk for preeclampsia. 3. My baby could come late because I am a teenager. 4. I am more likely to use birth control after I have this baby.

Answer: 1 Explanation: 1. Many teenage mothers drop out of school during their pregnancy and then are less likely to complete their schooling.

The nurse seeks to involve the adolescent father in the prenatal care of his girlfriend. What is the rationale for this nursing strategy? 1. Having the father more involved with the birth 2. Avoiding conflict between the adolescent father and pregnant teenager 3. Including his name of the birth certificate 4. Avoiding legal action by the adolescent fathers family

Answer: 1 Explanation: 1. Many young fathers genuinely want to be involved with their children and would have more contact and input if they could.

29) The client at 20 weeks gestation thinks she might have been exposed to a toxin at work that could affect fetal development. The client asks the nurse what organs might be affected at this point in pregnancy. What is the nurses best response? 1. The brain is developing now, and could be affected. 2. Because you are in the second trimester, there is no danger. 3. The internal organs like the heart and lungs could be impacted. 4. Its best to not worry about possible problems with your baby.

Answer: 1 Explanation: 1. Maximum brain growth and myelination are occurring at this point in fetal development.

19) The nurse in a fertility clinic is working with a woman who has been undergoing infertility treatment with clomiphene citrate. Which statement would the nurse expect the woman to make? 1. I feel moody so much of the time. 2. If this doesnt work, I think my husband will leave me. 3. This medication will guarantee a pregnancy. 4. My risk of twins or triplets is the same as for the general population.

Answer: 1 Explanation: 1. Mood swings are a side effect of clomiphene citrate.

34) The nurse teaching the expectant parents about the placenta also talks about the circulation and how the fetus gets its oxygen. She will include in this teaching which important fact? 1. The placenta functions as the lungs for the fetus. 2. The fetus obtains its oxygen from the amniotic fluid. 3. The fetus receives its oxygen by osmosis from the mothers bloodstream. 4. Fetal circulation delivers the highest amount of oxygen to the abdomen and lower body of the fetus.

Answer: 1 Explanation: 1. Most of the blood supply bypasses the fetal lungs because they do not carry out respiratory gas exchange. The placenta assumes the function of the fetal lungs by supplying oxygen and allowing the fetus to excrete carbon dioxide into the maternal bloodstream.

22) The nurse is working with a family who experienced the stillbirth of a son 2 months ago. Which statement by the mother would be expected? 1. I seem to keep crying for no reason. 2. The death of my son hasnt changed my life. 3. I have not visited my sons gravesite. 4. I feel happy all the time.

Answer: 1 Explanation: 1. Mourning may be manifested by certain behaviors and rituals, such as weeping, which help the person experience, accept, and adjust to the loss.

A Navajo client who is 36 weeks pregnant meets with a traditional healer as well as her physician. What does the nurse understand this to mean? 1. The client is seeking spiritual direction. 2. The client does not trust her physician. 3. The client will not adapt to mothering well. 4. The client is experiencing complications of pregnancy.

Answer: 1 Explanation: 1. Navajo clients are aware of the mind-soul connection, and might try to follow certain practices to have a healthy pregnancy and birth. Practices could include focus on peace and positive thoughts as well as certain types of prayers and ceremonies. A traditional healer may assist them.

19) The nurse is caring for a client pregnant with twins. Which statement indicates that the client needs additional information? 1. Because both of my twins are boys, I know that they are identical. 2. If my twins came from one fertilized egg that split, they are identical. 3. If I have one boy and one girl, I will know they came from two eggs. 4. It is rare for both twins to be within the same amniotic sac.

Answer: 1 Explanation: 1. Not all same-sex twins are identical or monozygotic, because fraternal, or dizygotic, twins can be the same gender or different genders.

25) The labor and delivery nurse is caring for a client whose labor is being induced due to fetal death in utero at 35 weeks gestation. In planning intrapartum care for this client, which nursing diagnosis is most likely to be applied? 1. Powerlessness 2. Urinary Elimination, Impaired 3. Coping: Family, Readiness for Enhanced 4. Skin Integrity, Impaired

Answer: 1 Explanation: 1. Powerlessness is commonly experienced by families who face fetal loss. Powerlessness is related to lack of control in current situational crisis.

23) The client with blood type A, Rh-negative, delivered yesterday. Her infant is blood type AB, Rh-positive. Which statement indicates that teaching has been effective? 1. I need to get RhoGAM so I dont have problems with my next pregnancy. 2. Because my baby is Rh-positive, I dont need RhoGAM. 3. If my baby had the same blood type I do, it might cause complications. 4. Before my next pregnancy, I will need to have a RhoGAM shot.

Answer: 1 Explanation: 1. Rh-negative mothers who give birth to Rh-positive infants should receive Rh immune globulin (RhoGAM) to prevent alloimmunization.

30) A pregnant client asks the nurse, What is this knuckle test that is supposed to tell whether my baby has a genetic problem? What does the nurse correctly explain? 1. In the first trimester, the nuchal translucency measurement is added to improve the detection rate for Down syndrome and trisomy 18. 2. You will need to ask the physician for an explanation. 3. It tests for hemophilia A or B. 4. It tests for Duchenne muscular dystrophy.

Answer: 1 Explanation: 1. Screening tests, such as nuchal translucency ultrasound are designed to gather information about the risk that the pregnancy could have chromosome abnormalities or open spina bifida.

36) The introduction of a new baby into the family is often the beginning of which of the following? 1. Sibling rivalry 2. Inconsistent childrearing 3. Toilet training 4. Weaning

Answer: 1 Explanation: 1. Sibling rivalry results from childrens fear of change in the security of their relationships with their parents.

After teaching a pregnant client about the effects of smoking on pregnancy, the nurse knows that the client needs further education when she makes which statement? 1. I am at increased risk for preeclampsia. 2. I am at increased risk for preterm birth. 3. I am at increased risk for placenta previa. 4. I am at increased risk for abruptio placentae.

Answer: 1 Explanation: 1. Smoking is not associated with increased risk for preeclampsia.

The nurse discussing different pregnancy programs explains that which of the following is critical to the success of an adolescent pregnancy-prevention program? 1. Including role models from the same cultural and racial backgrounds 2. Focusing on the adolescent female 3. Having short-term, informal programs available twice per year 4. Focusing on the expectations of the adolescents parents

Answer: 1 Explanation: 1. The National Campaign to Prevent Teen and Unplanned Pregnancys task forces found that the programs most effective at preventing teen pregnancy include models from the same cultural and racial backgrounds as the participants.

While completing the medical and surgical history during the initial prenatal visit, the 16-year-old primigravida interrupts with Why are you asking me all these questions? What difference does it make? Which statement would best answer the clients questions? 1. We ask these questions to detect anything that happened in your past that might affect the pregnancy. 2. We ask these questions to see whether you can have prenatal visits less often than most clients do. 3. We ask these questions to make sure that our paperwork and records are complete and up to date. 4. We ask these questions to look for any health problems in the past that might affect your parenting.

Answer: 1 Explanation: 1. The course of a pregnancy depends on a number of factors, including the past pregnancy history (if this is not a first pregnancy), prepregnancy health of the woman, presence of disease/illness states, family history, emotional status, and past health care.

The school nurse is planning a presentation on pregnancy for 13- and 14-year-olds who are currently pregnant. When planning the content of this presentation, what should the nurse keep in mind about these teens? 1. They are working on independence and autonomy. 2. They are no longer developing a sense of achievement. 3. They are confident in their own identity. 4. They are in control of their impulses.

Answer: 1 Explanation: 1. The developmental tasks of adolescence include developing an identity, gaining autonomy and independence, developing intimacy in a relationship, developing comfort with ones own sexuality, and developing a sense of achievement. Teens in early adolescence will not have achieved all of these tasks yet.

25) The pregnant client who is at 14 weeks gestation asks the nurse why the doctor used to call her baby an embryo, and now calls it a fetus. What is the best answer to this question? 1. A fetus is the term used from the ninth week of gestation and onward. 2. We call a baby a fetus when it is larger than an embryo. 3. An embryo is a baby from conception until the eighth week. 4. The official term for a baby in utero is really zygote.

Answer: 1 Explanation: 1. The fetal stage begins in the ninth week.

31) The prenatal clinic nurse has received four phone calls. Which client should the nurse call back first? 1. Pregnant woman at 28 weeks with history of asthma who is reporting difficulty breathing and shortness of breath 2. Pregnant woman at 6 weeks with a seizure disorder who is inquiring which foods are good folic acid sources for her 3. Pregnant woman at 35 weeks with a positive HBsAG who is wondering what treatment her baby will receive after birth 4. Pregnant woman at 11 weeks with untreated hyperthyroidism who is describing the onset of vaginal bleeding

Answer: 1 Explanation: 1. The goal of therapy is to prevent maternal exacerbations because even a mild exacerbation can cause severe hypoxia-related complications in the fetus.

The nurse is presenting a class to women who are currently pregnant or are planning pregnancy in the near future. Which client statement indicates that additional teaching is required? 1. The older a woman is when she conceives, the safer the pregnancy is. 2. Pregnant teens can have additional nutritional needs. 3. A woman whose sisters all had hypertension will be watched carefully. 4. Pregnancy may be more difficult to achieve in my 40s.

Answer: 1 Explanation: 1. The health risks associated with pregnancy vary by age. The risk for maternal death is significantly higher for women over age 35 and even higher for women age 40 and older. The incidence of low-birth-weight infants, preterm births, miscarriage, stillbirth, and perinatal morbidity and mortality is higher among women age 35 or older.

21) The kosher diet followed by many Jewish people forbids the eating of what foods? 1. Pig products and shellfish 2. Dairy products 3. All animal products 4. Dairy products and eggs

Answer: 1 Explanation: 1. The kosher diet followed by many Jewish people forbids the eating of pig products and shellfish. Certain cuts of meat from sheep and cattle are allowed, as are fish with fins and scales. In addition, many Jews believe that meat and dairy products should not be mixed or eaten at the same meal.

10) A woman at 7 weeks gestation is diagnosed with hyperemesis gravidarum. Which nursing diagnosis would receive priority? 1. Fluid Volume: Deficient 2. Cardiac Output, Decreased 3. Injury, Risk for 4. Nutrition, Imbalanced: Less than Body Requirements

Answer: 1 Explanation: 1. The newly admitted client with hyperemesis gravidarum has been experiencing excessive vomiting, and is in a fluid volume-deficit state.

10) At 32 weeks gestation, a woman is scheduled for a second non-stress test (following one she had at 28 weeks gestation). Which statement by the client would indicate an adequate understanding of this procedure? 1. I cant get up and walk around during the test. 2. Ill have an IV started before the test. 3. I can still smoke before the test. 4. I need to have a full bladder for this test.

Answer: 1 Explanation: 1. The purpose of the non-stress test is to determine the results of movement on fetal heart rate. The NST is typically performed with the woman in the semi-Fowlers position with a small pillow or blanket under the right hip to displace the uterus to the left.

A woman gave birth last week to a fetus at 18 weeks gestation after her first pregnancy. She is in the clinic for follow-up, and notices that her chart states she has had one abortion. The client is upset over the use of this word. How can the nurse best explain this terminology to the client? 1. Abortion is the obstetric term for all pregnancies that end before 20 weeks. 2. Abortion is the word we use when someone has miscarried. 3. Abortion is how we label babies born in the second trimester. 4. Abortion is what we call all babies who are born dead.

Answer: 1 Explanation: 1. The term abortion means a birth that occurs before 20 weeks gestation or the birth of a fetus-newborn who weighs less than 500 g. An abortion may occur spontaneously or it may be induced by medical or surgical means.

24) The nurse is present when a mother and her partner are told that their 35-week fetus has died. Which nursing intervention should the nurse perform first? 1. Encourage open communication with the family and the healthcare team. 2. Ask the family to withhold questions until the next day. 3. Request that another nurse come and care for this family. 4. Contact a local funeral home to help the family with funeral plans.

Answer: 1 Explanation: 1. The top priority for the nurse is to encourage open communications. The nurse functions as an advocate for the family in organizing interdisciplinary involvement, maintaining continuity of care, offering the opportunity for open communication, and ensuring that the familys wishes regarding their loss experience are honored.

The nurse is seeing prenatal clients in the clinic. Which client is exhibiting expected findings? 1. 12 weeks gestation, with fetal heart tones heard by Doppler fetoscope 2. 22 weeks gestation, client reports no fetal movement felt yet 3. 16 weeks gestation, fundus three finger breadths above umbilicus 4. Marked edema

Answer: 1 Explanation: 1. This is an expected finding because fetal heart tones should be heard by 12 weeks using a Doppler fetoscope.

30) The nurse educator is presenting a program to college students about factors that can cause congenital malformations. What should the nurse tell them? 1. The growing embryo is considered most vulnerable to hazardous agents during the first months of pregnancy. 2. Spontaneous abortion always occurs if the fetus is affected by a teratogen. 3. Potential teratogens can cause malformations of the heart, limbs, eyes, and other organ systems only in the second trimester. 4. Teratogen agents are primarily drugs.

Answer: 1 Explanation: 1. This is true. Because organs are formed primarily during embryonic development, the growing embryo is considered most vulnerable to hazardous agents during the first months of pregnancy.

7) After explaining how meiotic division occurs within the ovum, the nurse knows that the pregnant client understands when she makes what statement? 1. The second meiotic division is arrested until and unless the oocyte is fertilized. 2. Meiosis in the oocyte begins at puberty. 3. The first meiotic division continues when the female infant is born. 4. Fertilization does not take place in the secondary oocyte.

Answer: 1 Explanation: 1. This is true. The secondary oocyte moves into the metaphase stage of cell division, where its meiotic division is arrested until and unless the oocyte is fertilized.

11) The nurse is preparing for a postpartum home visit. The client has been home for a week, is breastfeeding, and experienced a third-degree perineal tear after vaginal delivery. The nurse should assess the client for which of the following? 1. Dietary intake of fiber and fluids 2. Dietary intake of folic acid and prenatal vitamins 3. Return of hemoglobin and hematocrit levels to baseline 4. Return of protein and albumin to predelivery levels

Answer: 1 Explanation: 1. This mother needs to avoid the risk of constipation. She might be hesitant to have a bowel movement due to anticipated pain from the perineal tear, and constipation will decrease the healing of the laceration.

25) The client at 14 weeks gestation has undergone a transvaginal ultrasound to assess cervical length. The ultrasound revealed cervical funneling. How should the nurse explain these findings to the client? 1. Your cervix has become cone-shaped and more open at the end near the baby. 2. Your cervix is lengthened, and you will deliver your baby prematurely. 3. Your cervix is short, and has become wider at the end that extends into the vagina. 4. Your cervix was beginning to open but now is starting to close up again.

Answer: 1 Explanation: 1. Transvaginal ultrasound can most accurately identify shortened cervical length and cervical funneling, which is a cone-shaped indentation in the cervical os indicating cervical insufficiency or risk of preterm labor.

30) A pregnant client is lactose intolerant. Which alternative food could this client eat to get sufficient calcium? 1. Turnip greens 2. Green beans 3. Cantaloupe 4. Nectarines

Answer: 1 Explanation: 1. Turnip greens are rich in calcium.

After telling a mother that her 13-year-old daughter is pregnant, the nurse would expect the mother to respond with which statement? 1. We had such high hopes for you. 2. But you have always been a happy child. 3. Ive always liked that boy. 4. This is just one of those things that happen.

Answer: 1 Explanation: 1. When an adolescent pregnancy is first revealed to the teens mother, the result is often anger, shame, or sorrow. The degree of negative response will be determined by the age of the teen, the family expectations for the teen, and the presence or absence of other teen pregnancies in the family or support network.

25) A client is concerned because she has been told her blood type and her babys are incompatible. What is the nurses best response? 1. This is called ABO incompatibility. It is somewhat common but rarely causes significant hemolysis. 2. This is a serious condition, and additional blood studies are currently in process to determine whether you need a medication to prevent it from occurring with a future pregnancy. 3. This is a condition caused by a blood incompatibility between you and your husband, but does not affect the baby. 4. This type of condition is very common, and the baby can receive a medication to prevent jaundice from occurring.

Answer: 1 Explanation: 1. When blood types, not Rh, are incompatible, it is called ABO incompatibility. The incompatibility occurs as a result of the maternal antibodies present in her serum and interaction between the antigen sites on the fetal RBCs.

12) A client with diabetes is receiving preconception counseling. The nurse will emphasize that during the first trimester, the woman should be prepared for which of the following? 1. The need for less insulin than she normally uses 2. Blood testing for anemia 3. Assessment for respiratory complications 4. Assessment for contagious conditions

Answer: 1 Explanation: 1. Women with diabetes often require less insulin during the first trimester.

6) The nurse evaluates the diet of a pregnant client and finds that it is low in zinc. The nurse knows that zinc intake should increase during pregnancy to promote protein metabolism. Which food should the nurse suggest in order to increase intake of zinc? 1. Shellfish 2. Bananas 3. Yogurt 4. Cabbage

Answer: 1 Explanation: 1. Zinc is found in greatest concentration in meats, shellfish, and poultry. Other good sources include whole grains and legumes.

10) The nurse is examining a pregnant woman in the third trimester. What skin changes should the nurse highlight as an alteration for the womans healthcare provider? 1. Linea nigra 2. Melasma gravidarum 3. Petechiae 4. Vascular spider nevi

Answer: 3 Explanation: 3. Petechiae are pinpoint red or purple spots on the skin. They are seen in hemorrhagic conditions.

12) A client at 32 weeks gestation comes to the clinic with urinary burning and frequency. The nurse explains that urinary tract infections are common in pregnancy due to which of the following? Select all that apply. 1. Ureteral atonia 2. Stasis of urine 3. Increased glomerular filtration rate 4. Increased renal plasma flow 5. Increased clearance of urea

Answer: 1, 2 Explanation: 1. The presence of amino acids and glucose in the urine in conjunction with the tendency toward ureteral atonia and stasis of urine in the ureters may increase the risk of urinary tract infection. 2. The presence of amino acids and glucose in the urine in conjunction with the tendency toward ureteral atonia and stasis of urine in the ureters may increase the risk of urinary tract infection.

A couple in their late 30s are pregnant for the first time. In evaluating the care delivered, the nurse assesses the mother for which of the following? Select all that apply. 1. Makes appropriate healthcare choices 2. Receives effective healthcare through the pregnancy 3. Has a partner who is not interested in child care 4. Cannot cope with her life change 5. Wishes to have amniocentesis done

Answer: 1, 2 Explanation: 1. Expected outcomes of nursing care for a pregnant couple over 35 include that the client and her partner are knowledgeable about the pregnancy and make appropriate healthcare choices. 2. Expected outcomes of nursing care for a pregnant couple over 35 include the client and her partner receive effective health care throughout the pregnancy, birth, and postpartum period.

The nurse is teaching about reproduction, and explains that which of the following are the purposes of meiosis? Select all that apply. 1. Produce gametes 2. Reduce the number of chromosomes 3. Introduce genetic variability 4. Produce cells for growth and development 5. Divide somatic cells into new cells with identical characteristics

Answer: 1, 2, 3 Explanation: 1. Meiosis is a special type of cell division by which diploid cells give rise to gametes (sperm and ova). 2. The cells contain half the genetic material of the parent cellonly 23 chromosomesthe haploid number of chromosomes. 3. During meiosis new combinations of cells are provided by the newly formed chromosomes; these combinations account for the wide variation of traits.

37) The clinic nurse is culturally sensitive when, while assessing the pregnant client, he asks about which of the following? Select all that apply. 1. The familys expectations of the healthcare system 2. Which cultural practices should be incorporated into care 3. Any alternative healer who should be consulted 4. Positive consequences of the clients healthcare beliefs 5. The clients giving up her practices and adopting the practices of the dominant culture

Answer: 1, 2, 3 Explanation: 1. The nurse needs to ask about the clients expectations of the healthcare system. 2. The nurse should ask about any cultural or spiritual practices that should be incorporated into care. 3. The culturally sensitive nurse will ask whether any alternative healer should be consulted about care.

11) The nurse is caring for a client who has just been informed of the demise of her unborn fetus. Which common cognitive responses to loss would the nurse anticipate? Note: Credit will be given only if all correct choices and no incorrect choices are selected. Select all that apply. 1. Denial and disbelief 2. Sense of unreality 3. Poor concentration 4. Palpitations 5. Loss of appetite

Answer: 1, 2, 3 Explanation: 1. Denial and disbelief are common cognitive responses to fetal loss. 2. A sense of unreality is a common cognitive response to fetal loss. 3. Poor concentration is a common cognitive response to loss.

During the initial prenatal visit, the nurse obtains a weight of 42 kg (92.4 lb). The nurse must further assess the client for information about which of the following? Select all that apply. 1. Eating habits 2. Foods regularly eaten 3. Income limitations 4. Blood pressure and pulse rate 5. Weight loss during pregnancy

Answer: 1, 2, 3 Explanation: 1. For a client whose weight is less than 100 lb, the nurse would obtain information on eating habits. 2. For a client whose weight is less than 100 lb, the nurse would obtain information on foods regularly eaten. 3. For a client whose weight is less than 100 lb, the nurse would obtain information on income limitations.

33) A pregnant asthmatic client is being seen for her initial prenatal visit. The nurse knows that the fetal implications of maternal asthma include which of the following? Note: Credit will be given only if all correct and no incorrect choices are selected. Select all that apply. 1. Prematurity 2. Low birth weight 3. Hypoxia with maternal exacerbation 4. Congenital anomalies from the medications 5. Perinatal transfer of the asthma

Answer: 1, 2, 3 Explanation: 1. One implication of maternal asthma is that the infant is at risk for prematurity. 2. One implication of maternal asthma is that the infant is at risk for low birth weight. 3. One implication of maternal asthma is that the infant is at risk for hypoxia if the mother has an exacerbation of her asthma.

The OB-GYN nurse works in a clinic with a culturally diverse group of clients whose specific actions during pregnancy are often determined by cultural beliefs. The nurse recognizes that these beliefs about pregnancy and childbirth fall into which categories? Select all that apply. 1. Prescriptive beliefs 2. Restrictive beliefs 3. Taboos 4. Cultural humility 5. Folk treatment beliefs

Answer: 1, 2, 3 Explanation: 1. Prescriptive beliefs or requirements describe expected behaviors. 2. Restrictive beliefs are stated negatively and limit behaviors. 3. Taboo beliefs refer to specific supernatural consequences.

What self-care measures would a nurse recommend for a client in her first trimester to reduce the discomfort of nausea and vomiting? Select all that apply. 1. Avoid odors or causative factors. 2. Have small but frequent meals. 3. Drink carbonated beverages. 4. Drink milk before arising in the morning. 5. Eat highly seasoned food.

Answer: 1, 2, 3 Explanation: 1. The nurse would recommend for a client in her first trimester to avoid odors and caustic factors to reduce the discomfort of nausea and vomiting. 2. The nurse would recommend for a client in her first trimester to have small but frequent meals to reduce the discomfort of nausea and vomiting. 3. The nurse would recommend for a client in her first trimester to drink carbonated beverages to reduce the discomfort of nausea and vomiting.

The prenatal period should be used to expose the prospective parents to up-to-date, evidence-based information about which of the following topics? Note: Credit will be given only if all correct and no incorrect choices are selected. Select all that apply. 1. Breastfeeding 2. Pain relief 3. Obstetric complications and procedures 4. Toddler care 5. Antepartum adjustment

Answer: 1, 2, 3 Explanation: 1. The prenatal period should expose prospective parents to up-to-date, evidence-based information about breastfeeding. 2. The prenatal period should expose prospective parents to up-to-date, evidence-based information about pain relief. 3. The prenatal period should expose prospective parents to up-to-date, evidence-based information about obstetric complications and procedures.

In planning a program to reduce teen pregnancy rates, the nurse uses an evidence-based approach. The nurse learns that more research is needed for which of the following? Select all that apply. 1. Teen pregnancy rate in the United States 2. Use of birth control by adolescents 3. Prevention of sexually transmitted infections 4. Long-active, reversible contraception is highly effective for teens 5. Intrauterine devices are safe and effective when used by adolescents

Answer: 1, 2, 3 Explanation: 1. The teen pregnancy rate in the United States requires more research. 2. The use of birth control by adolescents requires more research. 3. Preventing sexually transmitted infections in teenagers requires more research.

32) The nurse is working with a pregnant woman who has systemic lupus erythematosus (SLE). What does the nurse anticipate the infant might be born with? Note: Credit will be given only if all correct and no incorrect choices are selected. Select all that apply. 1. A tendency to bleed excessively 2. An increased chance of developing infections 3. A hemoglobin less than optimal for good health 4. Problems with vision 5. Hearing loss

Answer: 1, 2, 3 Explanation: 1. This is true, as the infant might be born with thrombocytopenia. 2. This is true, as the infant might be born with neutropenia. 3. This is true, as the infant might be born with anemia.

33) The partner of a pregnant client comes to the clinic with her. He complains to the nurse that he is experiencing different physical changes. The nurse determines he is experiencing couvade when he describes which symptoms? Select all that apply. 1. Fatigue 2. Increased appetite 3. Headache 4. Backache 5. High anxiety level

Answer: 1, 2, 3, 4 Explanation: 1. Couvade is demonstrated by increased fatigue in the partner. 2. Couvade is demonstrated by an increased appetite in the partner. 3. Couvade is demonstrated by the partners having headaches. 4. Couvade is demonstrated by the partners experiencing backache.

5) The nurse is working with a woman who abuses stimulants. The nurse is aware that the fetus is at risk for which of the following? Note: Credit will be given only if all correct and no incorrect choices are selected. Select all that apply. 1. Withdrawal symptoms 2. Cardiac anomalies 3. Sudden infant death syndrome 4. Being small for gestational age 5. Fetal alcohol syndrome

Answer: 1, 2, 3, 4 Explanation: 1. Infants born to mothers who abuse stimulants such as amphetamines can have withdrawal symptoms. 2. Infants born to mothers who abuse stimulants such as cocaine can be born with cardiac anomalies. 3. Infants born to mothers who abuse stimulants such as cocaine can have sudden infant death syndrome. 4. Infants born to mothers who abuse stimulants such as nicotine can be small for gestational age.

35) The nurse knows that a mother who has been treated for Beta streptococcus passes this risk on to her newborn. Risk factors for neonatal sepsis caused by Beta streptococcus include which of the following? Note: Credit will be given only if all correct choices and no incorrect choices are selected. Select all that apply. 1. Prematurity 2. Maternal intrapartum fever 3. Membranes ruptured for longer than 18 hours 4. A previously infected infant with GBS disease 5. An older mother having her first baby

Answer: 1, 2, 3, 4 Explanation: 1. Prematurity is a risk factor. 2. Maternal intrapartum fever is a risk factor. 3. Prolonged rupture of membranes is a risk factor. 4. A previously infected infant increases the risk.

2) The nurse educator is presenting a class on the different kinds of miscarriages. Miscarriages, or spontaneous abortions, are classified clinically into which of the following different categories? Note: Credit will be given only if all correct and no incorrect choices are selected. Select all that apply. 1. Threatened abortion 2. Incomplete abortion 3. Complete abortion 4. Missed abortion 5. Acute abortion

Answer: 1, 2, 3, 4 Explanation: 1. Unexplained cramping, bleeding, or backache indicates the fetus might be in jeopardy. This is a threatened abortion. 2. In an incomplete abortion, parts of the products of conception are retained, most often the placenta. 3. In a complete abortion, all the products of conception are expelled. The uterus is contracted and the cervical os may be closed. 4. In a missed abortion, the fetus dies in utero but is not expelled.

31) The nurse is teaching the significance of good nutrition on fetal development to a group of young women. What factors will the nurse discuss? Select all that apply. 1. Vitamins and folic acid are important before conception. 2. Maternal nutrition may predispose to the development of adult coronary heart disease, hypertension, and diabetes. 3. Glucose, amino acids, and fatty acids are of high importance. 4. Nutrition is important only during the first trimester. 5. Maternal nutrition can affect brain and neural tube development.

Answer: 1, 2, 3, 5 Explanation: 1. Vitamins and folic acid supplements taken before conception can reduce the incidence of neural tube defects. 2. This is true. Maternal nutrition may predispose to the development of adult coronary heart disease, hypertension, and diabetes. 3. Amino acids, glucose, and fatty acids are considered to be the primary dietary factors in brain growth. 5. Adequacy of the maternal environment is also important during the periods of rapid embryonic and fetal development. Maternal nutrition can affect brain and neural tube development.

During home care of a low-risk pregnant client, the nurse provides care by assessing which of the following? Select all that apply. 1. Urine 2. Weight 3. Diet 4. Pelvic measurements 5. Physical activity

Answer: 1, 2, 3, 5 Explanation: 1. Assessment of the clients urine can be done in the home setting. 2. Obtaining the clients weight can be done in the home setting. 3. Assessing the clients dietary intake can be done in the home setting. 5. The clients physical activity can be assessed in the home setting.

16) When blood pressure and other signs indicate that the preeclampsia is worsening, hospitalization is necessary to monitor the womans condition closely. At that time, which of the following should be assessed? Note: Credit will be given only if all correct choices and no incorrect choices are selected. Select all that apply. 1. Fetal heart rate 2. Blood pressure 3. Temperature 4. Urine color 5. Pulse and respirations

Answer: 1, 2, 3, 5 Explanation: 1. Determine the fetal heart rate along with blood pressure, or monitor continuously with the electronic fetal monitor if the situation indicates. 2. Determine blood pressure every 1 to 4 hours, or more frequently if indicated by medication or other changes in the womans status. 3. Determine temperature every 4 hours, or every 2 hours if elevated or if premature rupture of the membranes (PROM) has occurred. 5. Determine pulse rate and respirations along with blood pressure.

12) The nurse is counseling a group of first-trimester clients on diet increases that are necessary during pregnancy. Which information would be necessary to tell the pregnant women? Note: Credit will be given only if all correct choices and no incorrect choices are selected. Select all that apply. 1. An increase of protein is necessary to provide amino acids necessary for fetal development. 2. Protein contributes to the bodys overall energy metabolism. 3. The recommended protein during pregnancy is 70 grams each day. 4. The increased amount of protein that a pregnant woman needs is 15 grams a day. 5. The quality of protein is as important as the amount.

Answer: 1, 2, 3, 5 Explanation: 1. During pregnancy, the woman needs increased amounts of protein to provide amino acids for fetal development, blood volume expansion, and growth of other maternal tissues. 2. Protein contributes to overall energy metabolism. 3. The amount of protein recommended each day during pregnancy is 70 grams. 5. Plant proteins can meet a womans protein needs; however, more of a given protein may be needed because animal proteins are of higher quality.

29) Which of the following symptoms, if progressive, are indicative of CHF, the hearts signal of its decreased ability to meet the demands of pregnancy? Note: Credit will be given only if all correct and no incorrect choices are selected. Select all that apply. 1. Palpitations 2. Heart murmurs 3. Dyspnea 4. Frequent urination 5. Rales

Answer: 1, 2, 3, 5 Explanation: 1. Palpitations are indicative of CHF. 2. Heart murmurs are indicative of CHF. 3. Dyspnea is indicative of CHF. 5. Rales are indicative of CHF.

3) The nurse is teaching an infertile couple about the causes of infertility. The nurse tells them that infertility can be caused by which of the following? Select all that apply. 1. Immunological responses 2. Congenital anomalies 3. Patent fallopian tubes 4. Hypothyroidism 5. Favorable cervical mucus

Answer: 1, 2, 4 Explanation: 1. Immunological responses, such as antisperm antibodies, can cause infertility. 2. Congenital anomalies, such as a septate uterus, can cause infertility. 4. Hypothyroidism is a cause of infertility.

29) What of the following nursing interventions are appropriate when caring for the family experiencing a stillbirth? Note: Credit will be given only if all correct choices and no incorrect choices are selected. Select all that apply. 1. Use active listening techniques. 2. Avoid the use of clichs. 3. Avoid periods of silence. 4. Wrap the infant in a blanket before the parents see the infant. 5. Do not permit the parents of an infant with birth defects to hold the infant.

Answer: 1, 2, 4 Explanation: 1. It is important to allow the parents to verbalize their concerns. 2. The nurse can facilitate a healthy mourning process for the family by using active listening techniques and avoiding the use of clichs and platitudes. 4. The infant should be wrapped in a blanket to allow parents to see the infant before viewing any deformities.

A nurse working with adolescents recognizes risk factors for pregnancy include which of the following? Select all that apply. 1. Low socioeconomic status 2. A belief in invulnerability 3. A highly functional family 4. Physical, emotional, or sexual abuse 5. Accurate information about contraception

Answer: 1, 2, 4 Explanation: 1. Poverty is a major risk factor for adolescent pregnancy. 2. The sense of invulnerability and it wont happen to me thinking can lead adolescents to participate in sexual activity with an overly optimistic view of the outcomes of the risks associated with their actions. 4. Physical, emotional, or sexual abuse increases the risk of teenage pregnancy.

13) The nurse has written the nursing diagnosis Injury, Risk for for a diabetic pregnant client. Interventions for this diagnosis include which of the following? Note: Credit will be given only if all correct and no incorrect choices are selected. Select all that apply. 1. Assessment of fetal heart tones 2. Perform oxytocin challenge test, if ordered 3. Refer the client to a diabetes support group 4. Assist with the biophysical profile assessment 5. Develop an appropriate teaching plan

Answer: 1, 2, 4 Explanation: 1. Reassuring fetal heart rate variability and accelerations are interpreted as adequate placental oxygenation. 2. The nurse would perform oxytocin challenge test (OCT)/contraction stress test (CST) and non-stress tests as determined by physician. 4. The nurse assists the physician in performing a biophysical profile assessment.

Which of the following are specific culturally sensitive nursing considerations the nurse integrates into care of the pregnant client? Note: Credit will be given only if all correct and no incorrect choices are selected. Select all that apply. 1. Counseling about home remedies 2. Discussing components of a balanced diet 3. Refusing to discuss birthing choices 4. Encouraging use of support systems 5. Instructing the client to use no home remedies

Answer: 1, 2, 4 Explanation: 1. The culturally sensitive nurse should find out what medications and home remedies the client is using, and counsel the client regarding overall effects. 2. The culturally sensitive nurse will discuss the importance of a well-balanced diet during pregnancy with consideration of the clients cultural beliefs and practices. 4. Encouraging the use of support systems and spiritual aids that provide comfort for the mother is important in culturally sensitive care.

22) The nurse educator is teaching student nurses what a fetus will look like at various weeks of development. Which descriptions would be typical of a fetus at 20 weeks gestation? Select all that apply. 1. The fetus has a body weight of 435-465 g. 2. Nipples appear over the mammary glands. 3. The kidneys begin to produce urine. 4. Nails are present on fingers and toes. 5. Lanugo covers the entire body.

Answer: 1, 2, 4, 5 Explanation: 1. A fetus at 20 weeks gestation has a body weight of 435-465 g. 2. A fetus at 20 weeks gestation has nipples appear over the mammary glands. 4. A fetus at 20 weeks gestation nails are present on fingers and toes. 5. A fetus at 20 weeks gestation has lanugo that covers the entire body.

14) The nurse is talking with a couple who have been trying to get pregnant for 5 years. They are now at the fertility clinic seeking help. The nurse assesses their emotional responses as part of the workup. Which responses would the nurse expect to hear? Note: Credit will be given only if all correct choices and no incorrect choices are selected. Select all that apply. 1. Experiencing a sense of loss of status 2. Feelings of failure because they cannot make a baby 3. Healthy relationship with healthcare partners 4. Stress on the marital and sexual relationship 5. Feelings of frustration

Answer: 1, 2, 4, 5 Explanation: 1. The couple may experience feelings of loss of status and ambiguity as a couple. 2. Feelings of failure are common. 4. The couple may experience stress on the marital and sexual relationship. 5. Tests and treatments may heighten feelings of frustration or anger between partners.

28) The nurse is holding a class for newly pregnant couples, and discussing good and bad influences on their developing babies. Prenatal influences on the intrauterine environment include which of the following? Select all that apply. 1. The use of saunas or hot tubs 2. The use of drugs 3. The quality of the sperm or ovum 4. Maternal nutrition 5. Vitamins and folic acid

Answer: 1, 2, 4, 5 Explanation: 1. The use of saunas or hot tubs is associated with maternal hyperthermia. Studies of the effects of maternal hyperthermia during the first trimester have raised concern about possible spontaneous abortion, central nervous system (CNS) defects, and failure of neural tube closure. 2. The use of drugs can have teratogenic effects. 4. Maternal nutrition affects brain and neural tube development. 5. Vitamins and folic acid supplements taken before conception can reduce the incidence of neural tube defects.

3) The nurse is presenting a class on the pathophysiology of the different abortions. Some of the causes are which of the following? Note: Credit will be given only if all correct choices and no incorrect choices are selected. Select all that apply. 1. Chromosomal abnormalities 2. Insufficient or excessive hormonal levels 3. Sexual intercourse in the first trimester 4. Infections in the first trimester 5. Cervical insufficiency

Answer: 1, 2, 4, 5 Explanation: 1. Chromosomal defects are generally seen as spontaneous abortions during weeks 4 to 8. 2. Insufficient or excessive hormonal levels usually will result in spontaneous abortion by 10 weeks gestation. 4. Infectious and environmental factors may also be seen in first trimester pregnancy loss. 5. In late spontaneous abortion, the cause is usually a maternal factor, for example, cervical insufficiency or maternal disease, and fetal death may not precede the onset of abortion.

The nurse is planning an early-pregnancy class session on nutrition. Which information should the nurse include? Note: Credit will be given only if all correct choices and no incorrect choices are selected. Select all that apply. 1. Protein is important for fetal development. 2. Iron helps both mother and baby maintain the oxygen-carrying capacity of the blood. 3. Calcium prevents constipation at the end of pregnancy. 4. Zinc facilitates synthesis of RNA and DNA. 5. Vitamin A promotes development of the babys eyes.

Answer: 1, 2, 4, 5 Explanation: 1. During pregnancy, the woman needs increased amounts of protein to provide amino acids for fetal development. 2. Iron deficiency anemia is associated with an increased incidence of preterm birth, low-birth-weight infants, and maternal and infant mortality. 4. Zinc is involved in RNA and DNA synthesis, and milk production during lactation. 5. Vitamin A promotes healthy formation and development of the fetal eyes.

30) The nurse is evaluating the plan of care for a pregnant client with a heart disorder. The nurse concludes that the plan was successful when data indicate which of the following? Note: Credit will be given only if all correct and no incorrect choices are selected. Select all that apply. 1. The client gave birth to a healthy baby. 2. The client did not develop congestive heart failure. 3. The client developed thromboembolism. 4. The client identified manifestations of potential complications. 5. The client can identify her condition and its impact on her pregnancy, labor and birth, and postpartum period.

Answer: 1, 2, 4, 5 Explanation: 1. Giving birth to a healthy baby is an expected outcome of the pregnancy. 2. An expected outcome is that the woman does not develop congestive heart failure, thromboembolism, or infection. 4. An expected outcome is that the woman is able to identify potential complications and notify the healthcare provider. 5. The woman must be able to discuss her condition and its possible impact on her pregnancy, labor and birth, and the

Remedies for back pain in pregnancy that are supported by research evidence and may safely be taught to any pregnant woman by the nurse include which of the following? Note: Credit will be given only if all correct choices and no incorrect choices are selected. Select all that apply. 1. Pelvic tilt 2. Water aerobics 3. Sit-ups 4. Proper body mechanics 5. Maintaining good posture

Answer: 1, 2, 4, 5 Explanation: 1. The pelvic tilt can help restore proper body alignment and relieve back pain. 2. Exercise is an effective treatment for lower back pain. Exercise in water seems to provide benefits while being physically comfortable for expectant mothers. 4. The use of proper posture and good body mechanics throughout pregnancy is important. 5. Good posture is important because it allows more room for the stomach to function.

A pregnant client calls the clinic nurse to say she is worried about symptoms she is experiencing. The nurse advises the client to come immediately to the clinic because of which reported symptoms? Note: Credit will be given only if all correct and no incorrect choices are selected. Select all that apply. 1. Vaginal bleeding 2. Abdominal pain 3. Constipation 4. Epigastric pain 5. Blurring of vision

Answer: 1, 2, 4, 5 Explanation: 1. Vaginal bleeding can indicate abruptio placentae, placenta previa, or lesions of cervix or vagina, or it can be bloody show, and requires that the client be seen. 2. Abdominal pain can signal premature labor or abruptio placentae, and requires that the client be seen. 4. Epigastric pain must be evaluated, as it can indicate preeclampsia or ischemia in a major abdominal vessel. 5. Dizziness, blurring of vision, double vision, or spots before the eyes can indicate either hypertension and/or preeclampsia and requires the client be seen.

23) The client and her partner are carriers of sickle-cell disease. They are considering prenatal diagnosis with either amniocentesis or chorionic villus sampling (CVS). Which statements indicate that further teaching is needed on these two diagnostic procedures? Note: Credit will be given only if all correct choices and no incorrect choices are selected. Select all that apply. 1. Chorionic villus sampling carries a lower risk of miscarriage. 2. Amniocentesis can be done earlier in my pregnancy than CVS. 3. Neither test will conclusively diagnose sickle-cell disease in our baby. 4. The diagnosis comes sooner if we have CVS, not amniocentesis. 5. Amniocentesis is more accurate in diagnosis than the CVS.

Answer: 1, 2, 5 Explanation: 1. CVS has a risk of spontaneous abortion of 0.3% in cases. This rate is higher than second trimester amniocentesis. 2. CVS is performed in some medical centers for first trimester diagnosis after 9 completed weeks. Amniocentesis is performed between 15 and 20 weeks gestation. 5. Much like amniocentesis, chorionic villus sampling (CVS) is a procedure that is used to detect genetic, metabolic, and DNA abnormalities. CVS permits earlier diagnosis than can be obtained by amniocentesis.

17) The nurse is caring for a client who has just experienced a stillbirth. Which factors does the nurse recognize as potentially complicating the parents response to this loss? Note: Credit will be given only if all correct choices and no incorrect choices are selected. Select all that apply. 1. Unsupportive family 2. Adolescent mother 3. Strong religious faith 4. Open communication between the parents 5. Persistent denial of the situation

Answer: 1, 2, 5 Explanation: 1. Features of bereaved individuals circumstances that will put them at risk include an unsupportive or unavailable family. 2. With regard to age, adolescent parents probably pose the greatest challenge to nursing interventions. 5. Persistent denial hampers the grieving and healing processes. Page Ref: 947, 948

The nurse is planning a prenatal class for a group of pregnant adolescents. The nurse will establish which goals for the class? Note: Credit will be given only if all correct and no incorrect choices are selected. Select all that apply. 1. Provide anticipatory guidance 2. Prepare the participants for labor and birth 3. Avoid discussion of the clients conflicts 4. Provide community resources if asked 5. Help the participants develop coping skills

Answer: 1, 2, 5 Explanation: 1. Goals for a prenatal class designed for adolescents must include anticipatory guidance. 2. Goals for a prenatal class designed for adolescents must include preparing the participants for labor and the birth of their baby. 5. Goals for a prenatal class designed for adolescents must include helping the participants develop adequate coping skills.

21) During the history, the client admits to being HIV-positive and says she knows that she is about 16 weeks pregnant. Which statements made by the client indicate an understanding of the plan of care both during the pregnancy and postpartally? Note: Credit will be given only if all correct choices and no incorrect choices are selected. Select all that apply. 1. During labor and delivery, I can expect the zidovudine (ZDV) to be given in my IV. 2. After delivery, the dose of zidovudine (ZDV) will be doubled to prevent further infection. 3. My baby will be started on zidovudine (ZDV) for six weeks following the birth. 4. My babys zidovudine (ZDV) will be given in a cream form. 5. My baby will not need zidovudine (ZDV) if I take it during my pregnancy.

Answer: 1, 3 Explanation: 1. ART therapy generally it includes oral Zidovudine (ZDV) daily, IV ZDV during labor and until birth, and ZDV therapy for the infant for 6 weeks following birth. 3. ART therapy generally it includes oral Zidovudine (ZDV) daily, IV ZDV during labor and until birth, and ZDV therapy for the infant for 6 weeks following birth.

8) A couple request to see their stillborn infant. How should the nurse prepare the infant? Note: Credit will be given only if all correct choices and no incorrect choices are selected. Select all that apply. 1. Wrapping the infant in a blanket 2. Removing all blankets from the infant 3. Placing a hat on the infant 4. Removing any identification from the infant 5. Placing a diaper on the infant

Answer: 1, 3 Explanation: 1. After bathing, the infant should be placed in a suitable-sized gown and then wrapped in a blanket. Many parents will eventually remove the covering to inspect the infant; however, applying a covering allows them time to adjust to the appearance at their own pace. 3. A hat can be applied to cover birth defects. This allows the parents an opportunity to view the infant before seeing the birth defect.

The pregnant client states she does not want to take all these supplements. What recommendations could the nurse make for the client? Note: Credit will be given only if all correct choices and no incorrect choices are selected. Select all that apply. 1. Folic acid has been found to be essential for minimizing the risk of neural tube defects. 2. You do not have to take these supplements if you think you are healthy enough. 3. Most women do not have adequate intake of iron pre-pregnancy, and iron needs increase with pregnancy. 4. These medications do the same thing. I will call your physician to cancel one of your medications. 5. You should take the folic acid, but the vitamins are not that important.

Answer: 1, 3 Explanation: 1. An inadequate intake of folic acid has been associated with neural tube defects (NTDs) (e.g., spina bifida, anencephaly). 3. Iron is essential because many pregnant women do not have adequate intake of iron before pregnancy.

A pregnant client at 30 weeks gestation has had a steady rise in blood pressure. She is now 20 mmHg above her systolic baseline. The nurse advises her to immediately report which symptoms? Note: Credit will be given only if all correct and no incorrect choices are selected. Select all that apply. 1. Dizziness 2. Even a small amount of dependent edema 3. Spots before her eyes 4. Persistent nausea and vomiting 5. Vaginal spotting

Answer: 1, 3 Explanation: 1. Dizziness can be a sign of hypertension or preeclampsia, and should be reported immediately. 3. Spots before the eyes can be a sign of hypertension or preeclampsia, and should be reported immediately.

33) The client at 34 weeks gestation has been stabbed in the low abdomen by her boyfriend. She is brought to the emergency department for treatment. Which statements indicate that the client understands the treatment being administered? Note: Credit will be given only if all correct choices and no incorrect choices are selected. Select all that apply. 1. The baby needs to be monitored to check the heart rate. 2. My bowel has probably been lacerated by the knife. 3. I might need an ultrasound to look at the baby. 4. The catheter in my bladder will prevent urinary complications. 5. The IV in my arm will replace the amniotic fluid if it is leaking.

Answer: 1, 3 Explanation: 1. Ongoing assessments of trauma include evaluation of uterine tone, contractions and tenderness, fundal height, fetal heart rate, intake and output and other indicators of shock, normal postoperative evaluation in those women requiring surgery, determination of neurologic status, and assessment of mental outlook and anxiety level. 3. In cases of noncatastrophic trauma, where the mothers life is not directly threatened, fetal monitoring for 4 hours should be sufficient if there is no vaginal bleeding, uterine tenderness, contractions, or leaking amniotic fluid.

8) The nurse is teaching the students in their obstetric rotation about fertilization. What processes must the sperm undergo before fertilization can occur? 1. Capacitation and ovulation 2. Capacitation and the acrosomal reaction 3. Oogenesis and the acrosomal reaction 4. Gametogenesis and capacitation

Answer: 2 Explanation: 2. The sperm must undergo two processes before fertilization can occur: capacitation and the acrosomal reaction.

9) The nurse is assessing a client in the third trimester of pregnancy. What physiologic changes in the client are expected? Select all that apply. 1. The clients chest circumference has increased by 6 cm during the pregnancy. 2. The client has a narrowed subcostal angle. 3. The client is using thoracic breathing. 4. The client may have epistaxis. 5. The client has a productive cough.

Answer: 1, 3, 4 Explanation: 1. The chest increase compensates for the elevated diaphragm. 3. Breathing changes from abdominal to thoracic as pregnancy progresses. 4. Epistaxis (nosebleeds) may occur and are primarily the result of estrogen-induced edema and vascular congestion of the nasal mucosa.

28) The nurse in the OB-GYN clinic counsels a couple that in autosomal dominant inheritance, which of the following occur? Select all that apply. 1. An affected individual might have an affected parent. 2. The affected individual has a 75% chance of passing on the abnormality. 3. Males and females are equally affected by the gene. 4. A father can pass the defective gene to a son. 5. There are no variances in the genetic pattern for autosomal dominant disorders.

Answer: 1, 3, 4 Explanation: 1. The family pedigree usually shows multiple generations having the disorder. 3. Males and females are equally affected in autosomal dominant disorders. 4. In autosomal dominant disorders, the father can pass the defective gene to a son.

14) The nurse is instructing a pregnant client on her nutritional needs. The nurse tells the client that nutrition is needed for fetal development because fetal growth occurs in overlapping stages from increases of which of the following? Note: Credit will be given only if all correct and no incorrect choices are selected. Select all that apply. 1. Cell numbers 2. Cell membranes 3. Cell size and number 4. Cell size alone 5. Vitamin and mineral intake

Answer: 1, 3, 4 Explanation: 1. Fetal growth occurs by an increase in cell numbers in one stage. 3. Fetal growth occurs by an increase in cell size and number during one stage. 4. Fetal growth occurs by an increase in cell size alone during one stage.

29) A client at 10 weeks gestation has developed cholecystitis. If surgery is required, what is the safest time during pregnancy? 1. Immediately, before the fetus gets any bigger 2. Early in the second trimester 3. As close to term as possible 4. The risks are too high to do it anytime in pregnancy

Answer: 2 Explanation: 2. The early second trimester is the best time to operate because there is less risk of spontaneous abortion or early labor, and the uterus is not so large as to impinge on the abdominal field.

Which statements might a pregnant teenager be likely to make at her initial prenatal examination? Select all that apply. 1. I didnt know I could get pregnant the first time I had sex. 2. Several of my friends go to clinics to get contraception. 3. Its no big deal; two of my best friends have babies, too. 4. I was 13 years old when I had sex the first time. 5. My family and my boyfriend are really happy and supportive.

Answer: 1, 3, 4 Explanation: 1. Many factors contribute to the high teenage pregnancy rate, and one of them is lack of knowledge about conception. 3. Many factors contribute to the high teenage pregnancy rate, and one of them is a decreased social stigma of being a young and single mother. 4. Many factors contribute to the high teenage pregnancy rate, and one of them is a younger age at onset of sexual activity.

Postpartum nutritional status is determined primarily by assessing which of the following? Note: Credit will be given only if all correct choices and no incorrect choices are selected. Select all that apply. 1. Dietary history 2. Menstrual history 3. Mothers weight 4. Hemoglobin levels 5. Mothers height

Answer: 1, 3, 4 Explanation: 1. Postpartum nutritional status is determined by assessing the new mothers dietary history. 3. Postpartum nutritional status is determined by assessing the new mothers weight. 4. Postpartum nutritional status is determined by assessing the new mothers hemoglobin levels.

The nurse in the OB-GYN clinic is working with a client who is seeking her initial prenatal visit. The nurse will use the acronym TPAL to document the clients number of which of the following? Select all that apply. 1. Term infants born 2. Children living in the home 3. Pregnancies ending in abortion 4. Preterm infants born 5. Pregnancies that occurred

Answer: 1, 3, 4 Explanation: 1. T: number of term births the woman has experienced 3. A: number of pregnancies ending in either spontaneous or therapeutic abortion 4. P: number of preterm births

2) During a clients initial prenatal visit, the nurse must assess and document the clients current medical history, including which information? Select all that apply 1. Body mass index 2. Infections before the last menstrual period 3. Homeopathic or herbal medication use 4. Blood type and Rh factor 5. History of previous pregnancies

Answer: 1, 3, 4 Explanation: 1. The body mass index is an important part of the current medical history to be assessed and documented. 3. Homeopathic and herbal medication use is important for the nurse to assess and document in the current medical history. 4. The blood type must be assessed and documented in the current medical history, as must the Rh factor.

The nurse is explaining clinical pelvimetry to a client. The nurse explains that the anteroposterior diameters consist of which of the following? Note: Credit will be given only if all correct and no incorrect choices are selected. Select all that apply. 1. Diagonal conjugate 2. Transverse diameter 3. Conjugata vera 4. Obstetric conjugate 5. Oblique diameter

Answer: 1, 3, 4 Explanation: 1. The diagonal conjugate is a part of the anteroposterior diameter measurement. 3. The conjugata vera is a part of the anteroposterior diameter measurement. 4. The obstetric conjugate is a part of the anteroposterior diameter measurement.

2) The nurse understands that a clients pregnancy is progressing normally when what physiologic changes are documented on the prenatal record of a woman at 36 weeks gestation? Select all that apply. 1. The joints of the pelvis have relaxed, causing a waddling gait. 2. The cervix is firm and blue-purple in color. 3. The uterus vasculature contains one sixth of the total maternal blood volume. 4. Gastric emptying time is delayed, and the client complains of constipation and bloating. 5. Supine hypotension occurs when the client lies on her back.

Answer: 1, 3, 4, 5 Explanation: 1. The sacroiliac, sacrococcygeal, and pubic joints of the pelvis relax in the later part of the pregnancy, presumably as a result of hormonal changes. This often causes a waddling gait. 3. By the end of pregnancy, one sixth of the total maternal blood volume is contained within the vascular system of the uterus. 4. Gastric emptying time and intestinal motility are delayed, leading to frequent complaints of bloating and constipation, which can be aggravated by the smooth muscle relaxation and increased electrolyte and water reabsorption in the large intestine. 5. The enlarging uterus may exert pressure on the vena cava when the woman lies supine, causing a drop in blood pressure. This is called the vena caval syndrome, or supine hypotension.

5) The client in the first trimester of pregnancy questions the nurse about the causes of fetal death. The nurse explains that factors associated with perinatal loss include which of the following? Note: Credit will be given only if all correct choices and no incorrect choices are selected. Select all that apply. 1. Maternal diabetes 2. Paternal hypertension 3. Fetal chromosomal disorders 4. Maternal infections 5. Placental abnormalities

Answer: 1, 3, 4, 5 Explanation: 1. Fetal loss can be a result of a number of physiologic maladaptations, including maternal diabetes. 3. Chromosomal abnormalities can be associated with fetal loss. 4. Infections such as human parvovirus B19, syphilis, streptococcal infection, and Listeria can lead to fetal loss. 5. Placental abnormalities such as abruptio placentae and placenta previa can result in fetal death.

35) The OB-GYN nurse teaches a first-time pregnant client that functions of the amniotic fluid include which of the following? Select all that apply. 1. Allowing fetal freedom of movement 2. Releasing heat to control fetal temperature 3. Acting as an extension of fetal extracellular space 4. Providing a water source for the fetus to swallow 5. Acting as a wedge during labor

Answer: 1, 3, 5 Explanation: 1. Amniotic fluid permits fetal freedom of movement. 3. A primary function of amniotic fluid is to act as an extension of fetal extracellular space. 5. A primary function of amniotic fluid is to act as a wedge during labor.

15) Student nurses in their obstetrical rotation are learning about fertilization and implantation. The process of implantation is characterized by which statements? Select all that apply. 1. The trophoblast attaches itself to the surface of the endometrium. 2. The most frequent site of attachment is the lower part of the anterior uterine wall. 3. Between days 7 and 10 after fertilization, the zona pellucida disappears, and the blastocyst implants itself by burrowing into the uterine lining. 4. The lining of the uterus thins below the implanted blastocyst. 5. The cells of the trophoblast grow down into the uterine lining, forming the chorionic villi.

Answer: 1, 3, 5 Explanation: 1. During implantation, the trophoblast attaches itself to the surface of the endometrium for further nourishment. 3. Between days 7 and 10 after fertilization, the zona pellucida disappears, and the blastocyst implants itself by burrowing into the uterine lining and penetrating down toward the maternal capillaries until it is completely covered. 5. The cells of the trophoblast grow down into the thickened lining, forming the chorionic villi.

32) The clinic nurse is assessing how the prenatal client is meeting developmental tasks using Rubins tasks, including which of the following? Select all that apply. 1. Ensuring safe passage through pregnancy, labor, and birth. 2. Turning in on oneself to focus on the child. 3. Seeking commitment and acceptance of self as mother to the infant. 4. Completing the tasks of nesting at the appropriate time. 5. Seeking acceptance of the child by others.

Answer: 1, 3, 5 Explanation: 1. The tasks Rubin identified form the basis for a mutually gratifying relationship with the baby, and include ensuring safe passage through pregnancy, labor, and birth. 3. The tasks Rubin identified form the basis for a mutually gratifying relationship with the baby, and include seeking commitment and acceptance of self as mother. 5. The tasks Rubin identified form the basis for a mutually gratifying relationship with the baby, and include seeking acceptance of the child by others.

18) The nurse is caring for a client who finally conceived after several unsuccessful attempts at in vitro fertilization. The client has just been diagnosed with a perinatal loss. What should the nurses plan of care include? Note: Credit will be given only if all correct choices and no incorrect choices are selected. Select all that apply. 1. Giving accurate and honest information 2. Encouraging the couple to try right away to get pregnant again 3. Validating the many losses the client has experienced 4. Providing possible explanations for the fetal demise 5. Assessing where the client is in the grieving process, and communicating with compassion

Answer: 1, 3, 5 Explanation: 1. Families can cope with extreme situations when they are properly informed in an honest and forthright manner. 3. The nurse should be compassionate, give accurate and honest information, and validate the many losses incurred. 5. The nurse caring for a couple who has had a previous loss needs to be kind, compassionate, and patient.

A client at 34 weeks gestation complains about pyrosis. The nurse teaches the patient that approaches to relieve the pyrosis include which of the following? Select all that apply. 1. Eat small, frequent meals 2. Use high-sodium antacids 3. Avoid fried, fatty foods 4. Take sodium bicarbonate after meals 5. Do not lie down after eating

Answer: 1, 3, 5 Explanation: 1. Pyrosis (heartburn) can be relieved by eating small, more frequent meals. 3. Avoiding fatty, fried foods can relieve pyrosis. 5. Sitting up after meals will help decrease the pyrosis.

29) Women with eating disorders who become pregnant are at risk for a variety of complications including which of the following? Note: Credit will be given only if all correct choices and no incorrect choices are selected. Select all that apply. 1. Premature birth 2. Too many nutrients available for the fetus 3. Miscarriage 4. High birth weight 5. Perinatal mortality

Answer: 1, 3, 5 Explanation: 1. Risks to the mother and baby include premature birth. 3. Risks to the mother and baby include miscarriage. 5. Risks to the mother and baby include perinatal mortality.

14) The nurse is teaching a group of adolescents that sperm must undergo the process of capacitation in order to fertilize the ova. The characteristics of sperm that have undergone capacitation include which of the following? Note: Credit will be given only if all correct and no incorrect choices are selected. Select all that apply. 1. Ability to undergo an acrosomal reaction 2. Ability to block polyspermy 3. Ability to bind to the zona pellucida 4. Ability to release norepinephrine 5. Acquisition of hypermotility

Answer: 1, 3, 5 Explanation: 1. Sperm that undergo capacitation take on the ability to undergo the acrosomal reaction. 3. Sperm that undergo capacitation take on the ability to bind to the zona pellucida. 5. Sperm that undergo capacitation have the acquisition of hypermotility.

28) Which nursing interventions would be included in the plan of care for a family that has just been informed of a perinatal loss? Note: Credit will be given only if all correct choices and no incorrect choices are selected. Select all that apply. 1. Provide the parents with a private place and time to express their grief. 2. Offer reassurance that parents can have a subsequent successful pregnancy. 3. Allow the parents to participate in personal grief rituals. 4. Encourage interaction with other families. 5. Offer to give the family mementos of the infant such as footprints, crib card, and lock of hair.

Answer: 1, 3, 5 Explanation: 1. The couple with a known or suspected fetal demise should immediately be placed in a private room. When possible, the woman should be in a room that is farthest away from other laboring women. 3. The nurse should assist the couple in exploring their feelings and help them to make decisions about who will be present and what rituals will occur during and following the birth. 5. In a fetal demise, mementos are some of the few memories the parents have to provide them comfort after the death of their baby. Every effort should be made to offer as many quality mementos as possible, such as pictures and hand- or footprint molds and cards. Page Ref: 955, 957, 960

9) A client at 18 weeks gestation has been diagnosed with a hydatidiform mole. In addition to vaginal bleeding, which signs or symptoms would the nurse expect to see? Note: Credit will be given only if all correct choices and no incorrect choices are selected. Select all that apply. 1. Hyperemesis gravidarum 2. Diarrhea and hyperthermia 3. Uterine enlargement greater than expected 4. Polydipsia 5. Vaginal bleeding

Answer: 1, 3, 5 Explanation: 1. This is often seen in clients with hydatidiform mole. 3. This is a classic sign of hydatidiform mole. 5. This is a classic symptom of hydatidiform mole.

A client at 36 weeks gestation is complaining of dyspnea when lying flat. What is the clinical reason for this complaint? 1. Maternal hypertension 2. Fundal height 3. Hydramnios 4. Congestive heart failure

Answer: 2 Explanation: 2. The dyspnea is resulting from the pressure of the enlarging uterus on the diaphragm.

26) To answer a clients question about home pregnancy tests and their accuracy, the nurse must know that accuracy is affected by which of the following? Select all that apply. 1. Unclear directions 2. Unable to comprehend the directions 3. Blood in the specimen giving a false reading 4. Completing the test too late 5. Tagged antibodies becoming outdated

Answer: 1, 4 Explanation: 1. Women may not comprehend the HPT instructions, which can affect the accuracy results. 4. False-negative results typically occur when the test is completed too early or too late.

32) The clinic nurse assesses a newborn that is not progressing as expected. Genetic tests are ordered. The nurse explains to the parents that the laboratory tests to be done include which of the following? Select all that apply. 1. Chromosome analysis 2. Complete blood count 3. Phenylketonuria 4. Enzyme assay 5. Antibody titers

Answer: 1, 4, 5 Explanation: 1. Laboratory analysis includes chromosome analysis. 4. Laboratory analysis includes enzyme assay for inborn errors of metabolism. 5. Laboratory analysis includes antibody titers for infectious teratogens.

20) The nurse is planning a group session for clients who are beginning infertility evaluation. Which statements should be included in this session? Select all that apply. 1. Infertility can be stressful for a marriage. 2. The doctor will be able to tell why you have not conceived. 3. Your insurance will pay for the infertility treatments. 4. Keep communicating with one another through this process. 5. Support organizations can be helpful to deal with the emotional issues associated with infertility.

Answer: 1, 4, 5 Explanation: 1. Often an intact marriage will become stressed by the intrusive but necessary infertility procedures and treatments. 4. Communication is important; clients should communicate verbally and share feelings and support. 5. Referral to mental health professionals is helpful when the emotional issues become too disruptive in the couples relationship or life. Couples should be made aware of infertility support and education organizations, which may help meet some of these needs and validate their feelings.

The nurse is evaluating the expected outcomes of care for a pregnant adolescent. For the evaluation, the nurse will ask which of the following questions? Select all that apply. 1. Was a trusting relationship developed? 2. Did the adolescent explore financial options open to teen parents? 3. Did the adolescent make plans for continuing her education? 4. Can the adolescent problem-solve? 5. Did the adolescent follow recommendations?

Answer: 1, 4, 5 Explanation: 1. A trusting relationship must be developed with the adolescent client. 4. The adolescent must be able to appropriately problem-solve. 5. The adolescent must follow the recommendations of the healthcare team to deliver a healthy baby.

The nurses goals for prenatal classes may include which of the following? Note: Credit will be given only if all correct and no incorrect choices are selected. Select all that apply. 1. Increasing self-esteem 2. Preparing the participants for childrearing 3. Offering information on the risks of breastfeeding 4. Providing anticipatory guidance about pregnancy 5. Helping participants develop more adaptive coping skills

Answer: 1, 4, 5 Explanation: 1. Goals for prenatal classes would include increasing self-esteem. 4. Goals for prenatal class would include providing anticipatory guidance about pregnancy. 5. Goals for prenatal class would include helping participants develop more adaptive coping skills.

34) A client is admitted to the labor suite. It is essential that the nurse assess the womans status in relation to which infectious diseases? Note: Credit will be given only if all correct choices and no incorrect choices are selected. Select all that apply. 1. Chlamydia trachomatis 2. Rubeola 3. Varicella 4. Group B streptococcus 5. Acute pyelonephritis

Answer: 1, 4, 5 Explanation: 1. The infant may develop chlamydial pneumonia and Chlamydia trachomatis may be responsible for premature labor and fetal death. Chlamydial infection should be assessed. 4. Women may transmit GBS to their fetus in utero or during childbirth. GBS is a leading infectious cause of neonatal sepsis and mortality and should be assessed. 5. Acute pyelonephritis should be assessed as there is an increased risk of premature birth and intrauterine growth restriction (IUGR).

27) Which nursing diagnoses can apply to the couple experiencing a perinatal loss? Note: Credit will be given only if all correct choices and no incorrect choices are selected. Select all that apply. 1. Grieving related to the imminent loss of a child 2. Fear related to discomfort of labor and unknown outcome 3. Knowledge, Deficient related to lack of information about involution 4. Powerlessness related to lack of control in current situational crisis 5. Spiritual Distress, Risk for related to intense suffering secondary to unexpected fetal loss

Answer: 1, 4, 5 Explanation: 1. The nurse should anticipate that the family will experience the grieving process for the lost fetus. 4. The parents are faced with the sudden and unanticipated death of the unborn child, which occurred without any input or control on their part. 5. Spiritual distress is a common reaction of parents who experience an unanticipated loss. Page Ref: 954

23) Methods to increase fertility awareness include which of the following? Select all that apply. 1. Home assessment of cervical mucus 2. Pharmacologic agents 3. Therapeutic insemination 4. IVF 5. Basal body temperature (BBT) recordings

Answer: 1, 5 Explanation: 1. Methods to increase fertility awareness include home assessment of cervical mucus and basal body temperature (BBT) recordings. 5. Methods to increase fertility awareness include home assessment of cervical mucus and basal body temperature (BBT) recordings.

27) The pregnant teen who was prescribed prenatal vitamins at her initial prenatal visit states that she does not like to take them. How should the nurse respond? Note: Credit will be given only if all correct choices and no incorrect choices are selected. Select all that apply. 1. Folic acid has been found to be essential for minimizing the risk of neural tube defects. 2. You do not have to take these supplements if you think you are healthy enough. 3. These medications do the same thing. I will call your doctor to cancel one of your medications. 4. You can trust your doctor to know what you need. 5. You need the supplements because your dietary intake may not be adequate for fetal development.

Answer: 1, 5 Explanation: 1. The CDC estimates that most neural tube defects could be prevented if women followed folic acid supplementation recommendations before they know they are pregnant.

13) The true moment of fertilization occurs when what happens? 1. Cortical reaction occurs 2. Nuclei unite 3. Spermatozoa propel themselves up the female tract 4. Sperm surrounding the ovum release their enzymes

Answer: 2 Explanation: 2. The true moment of fertilization occurs as the nuclei unite. Their individual nuclear membranes disappear, and their chromosomes pair up to produce the diploid zygote.

30) The pregnant client at 14 weeks gestation is in the clinic for a regular prenatal visit. Her mother also is present. The grandmother-to-be states that she is quite uncertain about how she can be a good grandmother to this baby because she works full-time. Her own grandmother was retired, and was always available when needed by a grandchild. What is the nurses best response to this concern? 1. Dont worry. Youll be a wonderful grandmother. It will all work out fine. 2. What are your thoughts on what your role as grandmother will include? 3. As long as there is another grandmother available, you dont have to worry. 4. Grandmothers are supposed to be available. You should retire from your job.

Answer: 2 Explanation: 2. Although relationships with parents can be very complex, the expectant grandparents often become increasingly supportive of the expectant couple, even if conflicts previously existed. But it can be difficult for even sensitive grandparents to know how deeply to become involved in the childrearing process. In some areas, classes for grandparents provide information about changes in birthing and parenting practices.

18) Approximately 80% of anovulatory women have which condition? 1. Turner syndrome 2. Polycystic ovary syndrome (PCOS) 3. Klinefelter syndrome 4. Fragile X syndrome

Answer: 2 Explanation: 2. Approximately 80% of anovulatory women have polycystic ovary syndrome (PCOS), causing insulin resistance and hyperinsulinemia.

7) A client at 16 weeks gestation has a hematocrit of 35%. Her prepregnancy hematocrit was 40%. Which statement by the nurse best explains this change? 1. Because of your pregnancy, youre not making enough red blood cells. 2. Because your blood volume has increased, your hematocrit count is lower. 3. This change could indicate a serious problem that might harm your baby. 4. Youre not eating enough iron-rich foods like meat.

Answer: 2 Explanation: 2. Hemoglobin and hematocrit levels drop in early to mid-pregnancy as a result of pregnancy-associated hemodilution. Because the plasma volume increase (50%) is greater than the erythrocyte increase (25%), the hematocrit decreases slightly.

9) The nurse is reviewing assessment data from several different male clients. Which one should receive information about causes of infertility? 1. Circumcised client 2. Client with a history of premature ejaculation 3. Client with a history of measles at age 12 4. Client employed as an engineer

Answer: 2 Explanation: 2. Premature ejaculation is a possible cause of infertility.

3) A client who is in the second trimester of pregnancy tells the nurse that she has developed a darkening of the line in the midline of her abdomen from the symphysis pubis to the umbilicus. What other expected changes during pregnancy might she also notice? 1. Lightening of the nipples and areolas 2. Reddish streaks called striae on her abdomen 3. A decrease in hair thickness 4. Small purplish dots on her face and arms

Answer: 2 Explanation: 2. Striae, or stretch marks, are reddish, wavy, depressed streaks that may occur over the abdomen, breasts, and thighs as pregnancy progresses.

23) The adolescent client reports to the clinic nurse that her period is late, but that her home pregnancy test is negative. What should the nurse explain that these findings most likely indicate? 1. This means you are not pregnant. 2. You might be pregnant, but it might be too early for your home test to be accurate. 3. We dont trust home tests. Come to the clinic for a blood test. 4. Most people dont use the tests correctly. Did you read the instructions?

Answer: 2 Explanation: 2. This is a true statement. Most home pregnancy tests have low false-positive rates, but the false-negative rate is slightly higher. Repeating the test in a week is recommended.

21) A woman pregnant with twins asks the nurse about differences between identical and fraternal twins. The nurse explains that since it has been determined that she is having a boy and a girl, they are fraternal, and will have with of the following? 1. One placenta, two amnions, and two chorions 2. Two placentas, two amnions, and two chorions 3. Two placentas, one amnion, and two chorions 4. Two placentas, two amnions, one chorion

Answer: 2 Explanation: 2. This is the correct answer. Fraternal twins have two placentas, two amnions, and two chorions, however, the placentas sometimes fuse and look as if they are one.

20) The nurse is reviewing amniocentesis results. Which of the following would indicate that client care was appropriate? 1. The client who is Rh-positive received Rh immune globulin after the amniocentesis. 2. The client was monitored for 30 minutes after completion of the test. 3. The client began vaginal spotting before leaving for home after the test. 4. The client identified that she takes insulin before each meal and at bedtime.

Answer: 2 Explanation: 2. 20 to 30 minutes of fetal monitoring is performed after the amniocentesis.

A nurse examining a prenatal client recognizes that a lag in progression of measurements of fundal height from week to week and month to month could signal what condition? 1. Twin pregnancy 2. Intrauterine growth restriction 3. Hydramnios 4. Breech position

Answer: 2 Explanation: 2. A lag in progression of measurements of fundal height from month to month could signal intrauterine growth restriction (IUGR).

14) A client at 37 weeks gestation has a mildly elevated blood pressure. Her antenatal testing demonstrates three contractions in 10 minutes, no decelerations, and accelerations four times in 1 hour. What would this test be considered? 1. Positive non-stress test 2. Negative contraction stress test 3. Positive contraction stress test 4. Negative non-stress test

Answer: 2 Explanation: 2. A negative CST shows three contractions of good quality lasting 40 or more seconds in 10 minutes without evidence of late decelerations. This is the desired result.

5) Which of the following tests provides information about the fetal number? 1. Amniocentesis 2. Standard second-trimester sonogram 3. Beta hCG 4. Maternal serum alpha-fetoprotein

Answer: 2 Explanation: 2. A standard (comprehensive) second trimester sonogram provides the information about the fetus, placenta, and uterine conditions including fetal number.

6) A woman has just delivered a stillborn child at 26 weeks gestation. Which nursing action is appropriate at this time? 1. Remind the mother that she will be able to have another baby in the future. 2. Dress the infant in a gown and swaddle it in a receiving blanket. 3. Ask the woman whether she would like the doctor to prescribe a sedative for her. 4. Remove the baby from the delivery room as soon as possible.

Answer: 2 Explanation: 2. After bathing, the infant should be placed in a suitable-sized gown and then wrapped in a blanket.

0) The breastfeeding mother is concerned that her milk production has decreased. The nurse knows that further client teaching is needed based on which statement? 1. I am drinking a minimum of 8 to 10 glasses of liquid a day. 2. I have started cutting back on my protein intake. 3. At least three times a day, I drink a glass of milk. 4. My calorie intake is higher than during the pregnancy.

Answer: 2 Explanation: 2. An adequate protein intake is essential while breastfeeding because protein is an important component of breast milk.

A 25-year-old primigravida is at 20 weeks gestation. The nurse takes her vital signs and notifies the healthcare provider immediately because of which finding? 1. Pulse 88/minute 2. Rhonchi in both bases 3. Temperature 37.4 C (99.3 F) 4. Blood pressure 122/78

Answer: 2 Explanation: 2. Any abnormal breath sounds should be reported to the healthcare provider.

24) The nurse is presenting a class to pregnant clients. The nurse asks, The fetal brain is developing rapidly, and the nervous system is complete enough to provide some regulation of body function on its own, at which fetal development stage? It is clear that education has been effective when a participant makes which response? 1. The 17th-20th week 2. The 25th-28th week 3. The 29th-32nd week 4. The 33rd-36th week

Answer: 2 Explanation: 2. Between the 25th and 28th week, the brain is developing rapidly, and the nervous system is complete enough to provide some degree of regulation of body functions.

9) A newly diagnosed insulin-dependent type 1 diabetic with good blood sugar control is at 20 weeks gestation. She asks the nurse how her diabetes will affect her baby. What would the best explanation include? 1. Your baby could be smaller than average at birth. 2. Your baby will probably be larger than average at birth. 3. As long as you control your blood sugar, your baby will not be affected at all. 4. Your baby might have high blood sugar for several days.

Answer: 2 Explanation: 2. Characteristically, infants of mothers with diabetes are large for gestational age (LGA) as a result of high levels of fetal insulin production stimulated by the high levels of glucose crossing the placenta from the mother. Sustained fetal hyperinsulinism and hyperglycemia ultimately lead to excessive growth, called macrosomia, and deposition of fat.

6) The nurse is assessing a woman at 10 weeks gestation who is addicted to alcohol. The woman asks the nurse, What is the point of stopping drinking now if my baby probably has been hurt by it already? What is the best response by the nurse? 1. It wont help your baby, but you will feel better during your pregnancy if you stop now. 2. If you stop now, you and your baby have less chance of serious complications. 3. If you limit your drinking to once a week, your baby will be okay. 4. You might as well stop it now, because once your baby is born, youll have to give up alcohol if you plan on breastfeeding.

Answer: 2 Explanation: 2. Chronic abuse of alcohol can undermine maternal health by causing malnutrition, bone marrow suppression, increased incidence of infections, and liver disease. The effects of alcohol on the fetus may result in fetal alcohol spectrum disorders (FASD).

A 16-year-old is making her first prenatal visit to the clinic in her fourth month of pregnancy. What is the nurses first responsibility? 1. Contact the social worker. 2. Develop a trusting relationship. 3. Schedule the client for prenatal classes. 4. Teach the client about proper nutrition.

Answer: 2 Explanation: 2. Developing a trusting relationship with the pregnant adolescent is essential. Honesty and respect for the individual and a caring attitude promote the clients self-esteem.

What would the nurse do to accurately assess a pregnant clients food intake? 1. Assess her most recent laboratory values. 2. Ask her to complete a nutritional questionnaire. 3. Observe for signs of hunger. 4. Ask about her cooking facilities.

Answer: 2 Explanation: 2. Diet may be evaluated using a food frequency questionnaire, which lists common categories of foods and asks the woman how frequently in a day (or week) she consumes foods from the list.

13) Which of the following is important for the development of the central nervous system of the fetus? 1. Calcium and phosphorus 2. Essential fatty acids 3. Iron 4. Vitamin D

Answer: 2 Explanation: 2. Essential fatty acids are important for the development of the central nervous system of the fetus. Of particular interest are the omega-3 fatty acids and their derivatives.

12) A pregnant woman tells the nurse-midwife, Ive heard that if I eat certain foods during my pregnancy, the baby will be a boy. The nurse-midwifes response should explain that this is a myth, and that the sex of the baby is determined at what time? 1. At the time of ejaculation 2. At fertilization 3. At the time of implantation 4. At the time of differentiation

Answer: 2 Explanation: 2. Fertilization is the point at which the sex of the zygote is determined.

16) A client who is experiencing her first pregnancy has just completed the initial prenatal examination with a certified nurse-midwife. Which statement indicates that the client needs additional information? 1. Because we heard the babys heartbeat, I am undoubtedly pregnant. 2. Because I havent felt the baby move yet, we dont know whether Im pregnant. 3. My last period was 2 months ago, which means Im 2 months along. 4. The increased size of my uterus means that I am finally pregnant.

Answer: 2 Explanation: 2. Fetal movement is a subjective, or presumptive, change of pregnancy, and is not a reliable indicator in the early months of pregnancy.

6) During a class on genetics for pregnant families, the nurse is discussing the how the egg and sperm are formed before fertilization takes place. The nurse explains that these cells have only half the number of chromosomes, so when fertilization takes place, there will be the correct number. What is the process by which the egg and sperm are formed called? 1. Oogenesis 2. Gametogenesis 3. Meiosis 4. Spermatogenesis

Answer: 2 Explanation: 2. Gametogenesis is the process by which ovum and sperm are produced.

5) The nurse is assessing a client who has severe preeclampsia. What assessment finding should be reported to the physician? 1. Excretion of less than 300 mg of protein in a 24-hour period 2. Platelet count of less than 100,000/mm3 3. Urine output of 50 mL per hour 4. 12 respirations

Answer: 2 Explanation: 2. HELLP syndrome (hemolysis, elevated liver enzymes, and low platelet count) complicates 10% to 20% of severe preeclampsia cases and develops prior to 37 weeks gestation 50% of the time. Vascular damage is associated with vasospasm, and platelets aggregate at sites of damage, resulting in low platelet count (less than 100,000/mm3).

20) The client at 9 weeks gestation has been told that her HIV test was positive. The client is very upset, and tells the nurse, I didnt know I had HIV! What will this do to my baby? The nurse knows teaching has been effective when the client makes which statement? 1. I cannot take the medications that control HIV during my pregnancy, because they will harm the baby. 2. My baby can get HIV during the pregnancy and through my breast milk. 3. The pregnancy will increase the progression of my disease and will reduce my CD4 counts. 4. The HIV wont affect my baby, and I will have a low-risk pregnancy without additional testing.

Answer: 2 Explanation: 2. HIV transmission can occur during pregnancy and through breast milk; however, it is believed that the majority of all infections occur during labor and birth.

The nurse is working in a teen pregnancy clinic. In order to give the pregnant adolescent a role in her prenatal care, the nurse should allow the teen to do which of the following? 1. Choose the type of prenatal vitamin she takes 2. Measure and record her weight at each visit 3. Choose the schedule of her prenatal visits 4. Decide whether she wants her labor to be induced

Answer: 2 Explanation: 2. Having the client weigh herself and record her weight provides her the opportunity to participate in her own care plan.

7) A client is at 12 weeks gestation with her first baby. She has cardiac disease, class III. She states that she had been taking sodium warfarin (Coumadin), but her physician changed her to heparin. She asks the nurse why this was done. What should the nurses response be? 1. Heparin is used when coagulation problems are resolved. 2. Heparin is safer because it does not cross the placenta. 3. They are the same drug, but heparin is less expensive. 4. Coumadin interferes with iron absorption in the intestines.

Answer: 2 Explanation: 2. Heparin is safest for the client to take because it does not cross the placental barrier.

6) A woman is 16 weeks pregnant. She has had cramping, backache, and mild bleeding for the past 3 days. Her physician determines that her cervix is dilated to 2 centimeters, with 10% effacement, but membranes are still intact. She is crying, and says to the nurse, Is my baby going to be okay? In addition to acknowledging the clients fear, what should the nurse also say? 1. Your baby will be fine. Well start IV, and get this stopped in no time at all. 2. Your cervix is beginning to dilate. That is a serious sign. We will continue to monitor you and the baby for now. 3. You are going to miscarry. But you should be relieved because most miscarriages are the result of abnormalities in the fetus. 4. I really cant say. However, when your physician comes, Ill ask her to talk to you about it.

Answer: 2 Explanation: 2. If bleeding persists and abortion is imminent or incomplete, the woman may be hospitalized, IV therapy or blood transfusions may be started to replace fluid, and dilation and curettage (D&C) or suction evacuation is performed to remove the remainder of the products of conception.

31) Carbohydrates provide the bodys primary source of energy as well as fiber necessary for proper bowel functioning. If the carbohydrate intake is not adequate, the body will use which of the following for energy? 1. Iron 2. Protein 3. Vitamin C 4. Vitamin D

Answer: 2 Explanation: 2. If the carbohydrate intake is not adequate, the body uses protein for energy. Protein then becomes unavailable for growth needs.

10) A client is to receive fertility drugs prior to in vitro fertilization. What is the expected action of this medication? 1. Prolonging of the luteal phase 2. Stimulation of ovulation 3. Suppression of menstruation 4. Promotion of cervical mucus production

Answer: 2 Explanation: 2. In IVF, a womans ovaries are stimulated by a combination of medications, one or more oocytes are aspirated from her ovaries and fertilized in the laboratory, and then they are placed into her uterus after normal embryo development has begun.

The nurse is preparing a class for expectant fathers. Which information should the nurse include? 1. Siblings adjust readily to the new baby. 2. Sexual activity is safe for normal pregnancy. 3. The expectant mother decides the feeding method. 4. Fathers are expected to be involved in labor and birth.

Answer: 2 Explanation: 2. In a healthy pregnancy, there is no medical reason to limit sexual activity.

A Chinese woman who is 12 weeks pregnant reports to the nurse that ginseng and bamboo leaves help reduce her anxiety. How should the nurse respond to this client? 1. Advise the client to give up the bamboo leaves but to continue taking ginseng. 2. Advise the client to give up all herbal remedies. 3. Tell the client that her remedies have no scientific foundation. 4. Assess where the client obtains her remedy, and investigate the source.

Answer: 2 Explanation: 2. In some cases, the nurse might want to suggest remedies that may be more effective than herbal remedies. However, if the home remedy is not harmful, there is no reason for the nurse to ask a client to discontinue this practice.

3) The community nurse has identified that the mother who gave birth to a stillborn baby last week is an intuitive griever. Which behavior has the nurse encountered that would lead to this assessment? 1. The mother verbalized that her problem-solving skills have been helpful during this process. 2. The mother repeatedly talks about her thoughts, feelings, and emotions about losing her child. 3. The mother talks little about her experience, and appears detached and unaffected by the loss of her child. 4. The mother has asked close friends, co-workers, and relatives not to call or visit.

Answer: 2 Explanation: 2. Intuitive grievers tend to feel their way through the loss and seek emotional and psychosocial support.

33) The community nurse is planning care for a family that experienced the loss of twins at 20 weeks. Which of these steps should be part of the nurses care of this family? 1. Base care on the reactions of previous clients who experienced stillbirth. 2. Express the belief that the family will be able to get through this experience. 3. Encourage the couple to keep their feelings to themselves. 4. Honor the birth by reminding the couple that their babies are happy in heaven.

Answer: 2 Explanation: 2. Maintaining belief is defined as believing in the parents capacity to get through the event and face a future with meaning and it is one of the attributes of caring theory. Page Ref: 955

11) A couple who have sought fertility counseling have been told that the mans sperm count is very low. The nurse advises the couple that spermatogenesis is impaired when which condition occurs? 1. The vas deferens is ligated. 2. Male obesity is present. 3. The prostate gland is enlarged. 4. The flagella are segmented.

Answer: 2 Explanation: 2. Male obesity is associated with poor spermatogenesis and increased amount of time to conception.

16) A client presents to the antepartum clinic with a history of a 20-pound weight loss. Her pregnancy test is positive. She is concerned about gaining the weight back, and asks the nurse if she can remain on her diet. What is the nurses best response? 1. As long as you supplement your diet with the prenatal vitamin, the amount of weight you gain in pregnancy is not significant. 2. I understand that gaining weight after such an accomplishment might not look attractive, but weight gain during pregnancy is important for proper fetal growth. 3. Dieting during pregnancy is considered child neglect. 4. Excessive weight gain in pregnancy is due to water retention, so weight loss following birth will not be an issue.

Answer: 2 Explanation: 2. Maternal weight gain is an important factor in fetal growth and in infant birth weight. An adequate weight gain over time indicates an adequate caloric intake.

A pregnant client who swims 3-5 times per week asks the nurse whether she should stop this activity. What is the appropriate nursing response? 1. You should decrease the number of times you swim per week. 2. Continuing your exercise program would be beneficial. 3. You should discontinue your exercise program immediately. 4. You should consider a less strenuous type of exercise.

Answer: 2 Explanation: 2. Mild to moderate exercise is beneficial during pregnancy. Regular exerciseat least 30 minutes of moderate exercise daily or at least most days of the weekis preferred.

16) The nurse is caring for a client who experienced the birth of a stillborn son earlier in the day. The client is from a culture where a womans status is dominated by themes of motherhood and childrearing. What behavior would the nurse expect in this client? 1. Crying inconsolably 2. Expressing feelings of failure as a woman 3. Requesting family members to be present 4. Showing little emotion

Answer: 2 Explanation: 2. Mothers will often blame themselves, whether by commission or omission, particularly in cultures where a womans status is dominated by themes of motherhood and childrearing. Page Ref: 952

The clinic nurse is assisting with an initial prenatal assessment. The following findings are present: spider nevi present on lower legs; dark pink, edematous nasal mucosa; mild enlargement of the thyroid gland; mottled skin and pallor on palms and nail beds; heart rate 88 with murmur present. What is the best action for the nurse to take based on these findings? 1. Document the findings on the prenatal chart. 2. Have the physician see the client today. 3. Instruct the client to avoid direct sunlight. 4. Analyze previous thyroid hormone lab results.

Answer: 2 Explanation: 2. Mottling of the skin is indicative of possible anemia. These abnormalities must be reported to the healthcare provider immediately.

The nurse is assessing a primiparous client who indicates that her religion is Judaism. Why is this information is pertinent for the nurse to assess? 1. Religious and cultural background can impact what a client eats during pregnancy. 2. It provides a baseline from which to ask questions about the clients religious and cultural background. 3. Knowing the clients beliefs and behaviors regarding pregnancy is not important. 4. Clients sometimes encounter problems in their pregnancies based on what religion they practice.

Answer: 2 Explanation: 2. Nurses have an obligation to be aware of other cultures and develop a culturally sensitive plan of care to meet the needs of the childbearing woman and her family.

32) The nurse is caring for a client at 35 weeks gestation who has been critically injured in a shooting. Which statement by the paramedics bringing the woman to the hospital would cause the greatest concern? 1. Blood pressure 110/68, pulse 90. 2. Entrance wound present below the umbilicus. 3. Client is positioned in a left lateral tilt. 4. Clear fluid is leaking from the vagina.

Answer: 2 Explanation: 2. Penetrating trauma includes gunshot wounds and stab wounds. The mother generally fares better than the fetus if the penetrating trauma involves the abdomen as the enlarged uterus is likely to protect the mothers bowel from injury.

Which of the following is the most prevalent medical complication of pregnant adolescents? 1. Constipation 2. Preeclampsia-eclampsia 3. Heartburn 4. Rapid enlargement and sensitivity of breasts

Answer: 2 Explanation: 2. Preeclampsia-eclampsia is the most prevalent medical complication of pregnant adolescents and is typically characterized by high blood pressure, proteinuria, and edema.

27) Couples at risk for having a detectable single gene or chromosomal anomaly may wish to undergo which procedure? 1. Preimplantation genetic screening (PGS) 2. Preimplantation genetic diagnosis (PGD) 3. Intracytoplasmic sperm injection (ICSI) 4. Gamete intrafallopian transfer (GIFT)

Answer: 2 Explanation: 2. Preimplantation genetic diagnosis (PGD) is a term used when one or both genetic parents carry a gene mutation and testing is performed to determine whether that mutation or unbalanced chromosomal compliment has been passed to the oocyte or embryo.

22) A woman confides to the nurse that she has pica. What alternative could the nurse suggest to the client? 1. Replace laundry starch with salt. 2. Replace ice with frozen fruit pops. 3. Replace soap with cream cheese. 4. Replace soil with nuts.

Answer: 2 Explanation: 2. Some women are able to switch to frozen fruit pops instead of ice.

The nurse begins a prenatal assessment on a 25-year-old primigravida at 20 weeks gestation and immediately contacts the healthcare provider because of which finding? 1. Pulse 88/minute 2. Respirations 30/minute 3. Temperature 37.4 C (99.3 F) 4. Blood pressure 118/82

Answer: 2 Explanation: 2. Tachypnea is not a normal finding and requires medical care.

24) The nurse anticipates that the physician will most likely order a cervicovaginal fetal fibronectin test for which client? 1. The client at 34 weeks gestation with gestational diabetes 2. The client at 32 weeks gestation with regular uterine contractions 3. The client at 37 weeks multi-fetal gestation 4. The client at 20 weeks gestation with ruptured amniotic membranes

Answer: 2 Explanation: 2. The absence of cervicovaginal fFN between 20 and 34 weeks gestation has been shown to be a strong predictor of a woman not experiencing preterm birth due to spontaneous preterm labor or premature rupture of membranes. Positive findings indicate a 99% probability of birth within the next 2 weeks.

A nurse working in an adolescent clinic that deals with birth control, pregnancy, and referrals for adoption has a number of clients in early adolescence. Clients in that age group are how old? 1. 11-16 years old 2. 11-14 years old 3. 14 and 15 years old 4. 12-15 years old

Answer: 2 Explanation: 2. The age range for early adolescence is age 14 and under.

16) Of all the clients who have been scheduled to have a biophysical profile, the nurse should check with the physician and clarify the order for which client? 1. A gravida with intrauterine growth restriction 2. A gravida with mild hypotension of pregnancy 3. A gravida who is postterm 4. A gravida who complains of decreased fetal movement for 2 days

Answer: 2 Explanation: 2. The biophysical profile is used when there is a risk of placental and/or fetal compromise. The gravida with mild hypotension will need to be monitored more closely throughout the pregnancy, but is not a candidate at present for a biophysical profile.

18) On examination of the prenatal client, the nurse is aware that she will assess for a bluish pigmentation of the vagina. What is this objective (probable) sign of pregnancy also known as? 1. Hegar sign 2. Chadwick sign 3. Nightingale sign 4. Goodell sign

Answer: 2 Explanation: 2. The blue-purple discoloration of the cervix is Chadwick sign.

The nurse receives a phone call from a client who claims she is pregnant. The client reports that she has regular menses that occur every 28 days and last 5 days. The first day of her last menses was April 10. What would the clients estimated date of delivery (EDD) be if she is pregnant? 1. Nov. 13 2. Jan. 17 3. Jan. 10 4. Dec. 3

Answer: 2 Explanation: 2. The due date is Jan. 17. Nageles rule is to add 7 days to the last menstrual period and subtract 3 months. The last menstrual period is April 10, therefore Jan. 17 is the EDD.

The nurse working in an outpatient obstetric clinic assesses four primigravida clients. Which client findings would the nurse tell the physician about? 1. 17 weeks gestation and client denies feeling fetal movement 2. 24 weeks gestation and fundal height is at the umbilicus 3. 4-6 weeks gestation and softening of the cervix 4. 34 weeks gestation and complains of hemorrhoidal pain

Answer: 2 Explanation: 2. The fundal height at 24 weeks should be 24 cm. The fundal height is usually at the umbilicus at 20-22 weeks.

16) Which of the following may be the main presenting symptom of iron deficiency anemia? 1. Frequent urination 2. Fatigue 3. Nausea 4. Headaches

Answer: 2 Explanation: 2. The main presenting symptom of iron deficiency anemia may be fatigue.

10) How does the nurse consider the spiritual needs of a couple experiencing a fetal loss? 1. Explaining the fetal loss in terms of the nurses own religious beliefs 2. Providing an atmosphere of acceptance regarding the couples spiritual rites 3. Referring the couple to the hospital chaplain at discharge 4. Informing the couple of religious rituals that have helped other couples to cope with fetal loss

Answer: 2 Explanation: 2. The nurse can facilitate the spiritual needs of the couple by providing an atmosphere of acceptance regarding spiritual rites and encouraging the couples use of spiritual writings, prayers, and observances.

12) An infertile couple confides in the nurse at the infertility clinic that they feel overwhelmed with the decisions facing them. Which nursing strategy would be most appropriate? 1. Refer them to a marriage counselor. 2. Provide them with information and instructions throughout the diagnostic and therapeutic process. 3. Express concern and caring. 4. Inquire about the names they have chosen for their baby.

Answer: 2 Explanation: 2. The nurse can provide comfort to couples by offering a sympathetic ear, a nonjudgmental approach, and appropriate information and instruction throughout the diagnostic and therapeutic processes.

10) A 26-year-old client is 28 weeks pregnant. She has developed gestational diabetes. She is following a program of regular exercise, which includes walking, bicycling, and swimming. What instructions should be included in a teaching plan for this client? 1. Exercise either just before meals or wait until 2 hours after a meal. 2. Carry hard candy (or other simple sugar) when exercising. 3. If your blood sugar is 120 mg/dL, eat 20 g of carbohydrate. 4. If your blood sugar is more than 120 mg/dL, drink a glass of whole milk.

Answer: 2 Explanation: 2. The nurse should advise her to carry a simple sugar such as hard candy because of the possibility of exercise-induced hypoglycemia.

20) A 16-year-old pregnant client is seen at her 10-weeks-gestation visit. She tells the nurse that she felt the baby move that morning. What response by the nurse is appropriate? 1. That is very exciting. The baby must be very healthy. 2. Would you please describe what you felt for me? 3. That is impossible. The baby is not big enough yet. 4. Would you please let me see whether I can feel the baby?

Answer: 2 Explanation: 2. The nurse should ask the client to describe what she felt, as 10 weeks gestation is too early to feel fetal movement.

31) The nurse is working with a laboring woman who has a known intrauterine fetal demise. To facilitate the familys acceptance of the fetal loss, after delivery the nurse should do which of the following? 1. Encourage the parents to look at the infant from across the room. 2. Offer the parents the choice of holding the infant in their arms. 3. Take the infant to the morgue immediately. 4. Call family members and inform them of the birth.

Answer: 2 Explanation: 2. The nurse should offer the couple the opportunity to see and hold the infant and reassure the couple that any decision they make for themselves is the right one.

34) Nurses who are interacting with expectant families from a different culture or ethnic group can provide more effective, culturally sensitive nursing care by doing what? 1. Recognizing that ultimately it is the familys right to make a womans healthcare choices. 2. Obtaining a medical interpreter of the language the client speaks. 3. Evaluating whether the clients healthcare beliefs have any positive consequences for her health. 4. Accepting personal biases, attitudes, stereotypes, and prejudices.

Answer: 2 Explanation: 2. The nurse should provide for the services of an interpreter if language barriers exist.

The pregnant client has asked the nurse what kinds of medications cause birth defects. Which statement would best answer this question? 1. Birth defects are very rare. Dont worry; your doctor will watch for problems. 2. To be safe, dont take any medication without talking to your doctor. 3. Too much vitamin C is one of the most common issues. 4. Almost all medications will cause birth defects in the first trimester.

Answer: 2 Explanation: 2. The nurse should remind the client of the need to check with her caregiver about medications. If a woman has taken a drug in category D or X, she should be informed of the risks associated with that drug and of her alternatives.

32) During a prenatal examination, an adolescent client asks, How does my baby get air? What correct information would the nurse give? 1. The lungs of the fetus carry out respiratory gas exchange in utero similar to what an adult experiences. 2. The placenta assumes the function of the fetal lungs by supplying oxygen and allowing the excretion of carbon dioxide into your bloodstream. 3. The blood from the placenta is carried through the umbilical artery, which penetrates the abdominal wall of the fetus. 4. The fetus is able to obtain sufficient oxygen due to the fact that your hemoglobin concentration is 50% greater during pregnancy.

Answer: 2 Explanation: 2. The placenta assumes the function of the fetal lungs by supplying oxygen and allowing the excretion of carbon dioxide into the maternal bloodstream.

27) A prenatal educator is asking a partner about normal psychological adjustment of an expectant mother during the second trimester of pregnancy. Which answer by the partner would indicate a typical expectant mothers response to pregnancy? 1. She is very body-conscious, and hates every little change. 2. She daydreams about what kind of parent she is going to be. 3. I havent noticed anything. I just found out she was pregnant. 4. She has been having dreams at night about misplacing the baby.

Answer: 2 Explanation: 2. The second trimester brings increased introspection and consideration of how she will parent. She might begin to get furniture and clothing as concrete preparation, and feels movement and is aware of the fetus and incorporates it into herself.

11) The prenatal clinic nurse is caring for a client with hyperemesis gravidarum at 14 weeks gestation. The vital signs are: blood pressure 95/48, pulse 114, respirations 24. Which order should the nurse implement first? 1. Weigh the client. 2. Give 1 liter of lactated Ringers solution IV. 3. Administer 30 mL Maalox (magnesium hydroxide) orally. 4. Encourage clear liquids orally.

Answer: 2 Explanation: 2. The vital signs indicate hypovolemia from dehydration, which leads to hypotension and increased pulse rate. Giving this client a liter of lactated Ringers solution intravenously will reestablish vascular volume and bring the blood pressure up, and the pulse and respiratory rate down.

The prenatal clinic nurse is designing a new prenatal intake information form for pregnant clients. Which question is best to include on this form? 1. Where was the father of the baby born? 2. Do genetic diseases run in the family of the babys father? 3. What is the name of the babys father? 4. Are you married to the father of the baby?

Answer: 2 Explanation: 2. This question has the highest priority because it gets at the physiologic issue of inheritable genetic diseases that might directly impact the baby.

30) The nurse is caring for a 15-year-old who just delivered a 32-weeks-gestation stillborn infant with numerous defects. In caring for this client, the nurse knows which of the following? 1. The client will likely do no grieving, as she is so young and the pregnancy was probably a mistake in any case. 2. Adolescents have a sense of invulnerability, an It cant happen to me mentality. 3. The clients mother will handle her daughters grief, so the nurse doesnt need to be concerned. 4. The nurse will remove the baby before the client sees it.

Answer: 2 Explanation: 2. Though adolescents have a mature concept of death, it is often clouded by their sense of invulnerability, an It cant happen to me mentality. Page Ref: 947

4) The nurse is anticipating the arrival of a couple in the labor unit. It has been determined that the 37-week fetus has died in utero from unknown causes. What should the nurse include in the plan of care for this couple? 1. Allow the couple to adjust to the labor unit in the waiting area. 2. Place the couple in a labor room at the end of the hall with an empty room next door. 3. Encourage the father to go home and rest for a few hours. 4. Contact the mothers emergency contact person and explain the situation.

Answer: 2 Explanation: 2. Upon arrival to the facility, the couple with a known or suspected fetal demise should immediately be placed in a private room. When possible, the woman should be in a room that is farthest away from other laboring women.

26) The school nurse is planning a class about nutrition for pregnant teens, several of whom have been diagnosed with iron-deficiency anemia. In order to increase iron absorption, the nurse would encourage the teens to consume more of what beverage? 1. Gatorade 2. Orange juice 3. Milk 4. Green tea

Answer: 2 Explanation: 2. Vitamin C is found in citrus fruits and juices, and is known to enhance the absorption of iron from meat and non-meat sources.

A pregnant teenage client is diagnosed with iron-deficiency anemia. Which nutrient should the nurse encourage her to take to increase iron absorption? 1. Vitamin A 2. Vitamin C 3. Vitamin D 4. Vitamin E

Answer: 2 Explanation: 2. Vitamin C is known to enhance the absorption of iron from meat and nonmeat sources.

22) A woman is 32 weeks pregnant. She is HIV-positive but asymptomatic. The nurse knows what would be important in managing her pregnancy and delivery? 1. An amniocentesis at 30 and 36 weeks 2. Weekly non-stress testing beginning at 32 weeks gestation 3. Application of a fetal scalp electrode as soon as her membranes rupture in labor 4. Administration of intravenous antibiotics during labor and delivery

Answer: 2 Explanation: 2. Weekly non-stress testing (NST) is begun at 32 weeks gestation and serial ultrasounds are done to detect IUGR.

2) The nurse is doing preconception counseling with a 28-year-old woman with no prior pregnancies. Which statement made by the client indicates to the nurse that the client has understood the teaching? 1. I can continue to drink alcohol until I am diagnosed as pregnant. 2. I need to stop drinking alcohol completely when I start trying to get pregnant. 3. A beer once a week will not damage the fetus. 4. I can drink alcohol while breastfeeding because it doesnt pass into breast milk.

Answer: 2 Explanation: 2. Women should discontinue drinking alcohol when they start to attempt to become pregnant due to possible effects of alcohol on the fetus.

25) Women with HIV should be evaluated and treated for other sexually transmitted infections and for what condition occurring more commonly in women with HIV? 1. Syphilis 2. Toxoplasmosis 3. Gonorrhea 4. Herpes

Answer: 2 Explanation: 2. Women with HIV should be evaluated and treated for other sexually transmitted infections and for conditions occurring more commonly in women with HIV, such as tuberculosis, cytomegalovirus, toxoplasmosis, and cervical dysplasia.

2) The nurse is preparing a class on reproduction. What is the cell division process that results in two identical cells, each with the same number of chromosomes as the original cell called? 1. Meiosis 2. Mitosis 3. Oogenesis 4. Gametogenesis

Answer: 2 Explanation: 2. Mitosis results in the production of diploid body (somatic) cells, which are exact copies of the original cell.

22) The nurse recognizes that subjective pregnancy changes such as amenorrhea can be caused by which conditions? Select all that apply. 1. Goodell sign 2. Anemia 3. Pseudocyesis 4. Thyroid dysfunction 5. Fetal heartbeat

Answer: 2, 3, 4 Explanation: 2. Anemia can cause amenorrhea, and is a subjective sign of pregnancy. 3. Pseudocyesis (intense desire for pregnancy) can cause amenorrhea. 4. Thyroid dysfunction can cause amenorrhea, and is a subjective sign of pregnancy.

In planning a conference for adolescents, the nurse researches the most common socioeconomic and cultural factors that contribute to adolescent pregnancy. Which true statements should be included in this conference? Note: Credit will be given only if all correct choices and no incorrect choices are selected. Select all that apply. 1. Economic status has a relatively low impact on an adolescents sexual activity. 2. Nearly three quarters of all teens ages 15-19 have had sex at least once. 3. Poor self-esteem contributes to adolescent pregnancy. 4. 38% percent of sexually active students use a condom. 5. Sexual abuse is not an issue with teen mothers.

Answer: 2, 3 Explanation: 2. By 19 years of age, 70% of all teens have had intercourse. 3. This is a true statement. Poor self-esteem is a major contributing factor in adolescent pregnancy.

1) The nurse is planning an in-service presentation about perinatal loss. Which statements should the nurse include in this presentation? Note: Credit will be given only if all correct choices and no incorrect choices are selected. Select all that apply. 1. Perinatal loss refers to third-trimester fetal death in utero. 2. Perinatal loss occurs more frequently in assisted reproduction. 3. Perinatal loss rates have declined in the United States over the past few years. 4. Perinatal loss includes 25% of stillbirths occurring before the onset of labor. 5. Perinatal loss rarely causes an emotional problem for the family.

Answer: 2, 3 Explanation: 2. Pregnancies conceived by in vitro fertilization have higher rates of pregnancy loss and pregnancy complications. 3. Perinatal loss in industrialized countries has declined in recent years as early diagnosis of congenital anomalies and advances in genetic testing techniques have increased the use of elective termination.

27) The client presents to the clinic for an initial prenatal examination. She asks the nurse whether there might be a problem for her baby because she has type B Rh-positive blood and her husband has type O Rh-negative blood, or because her sisters baby had ABO incompatibility. What is the nurses best answer? Note: Credit will be given only if all correct choices and no incorrect choices are selected. Select all that apply. 1. Your baby would be at risk for Rh problems if your husband were Rh-negative. 2. Rh problems only occur when the mother is Rh-negative and the father is not. 3. ABO incompatibility occurs only after the baby is born. 4. We dont know for sure, but we can test for ABO incompatibility. 5. Your husbands being type B puts you at risk for ABO incompatibility.

Answer: 2, 3 Explanation: 2. Rh incompatibility is a possibility when the mother is Rh-negative and the father is Rh-positive. 3. ABO incompatibility is limited to type O mothers with a type A or B fetus and occurs after the baby is born.

10-19, 36

http://testbankgo.eu/olds-maternal-newborn-nursing-womens-health-across-the-lifespan-10th-edition-test-bank/

The nurse is preparing an in-service presentation for a group of middle-school nurses. Which statements by the nurse would indicate that the middle-school nurse understood the role of culture in adolescent pregnancy? Select all that apply. 1. Studies show that 85% of teen mothers are middle class, and give birth to gain adult status. 2. Teenage pregnancy is the leading reason why adolescent women drop out of school. 3. Teens of color are more likely to become pregnant. 4. Intelligence and academic ability are positively associated with delayed sexual activity, greater use of contraception, and lower rates of pregnancy. 5. Over 50% of adolescents who have had an abortion or recent birth become pregnant again within two years.

Answer: 2, 3, 4 Explanation: 2. An estimated 30% to 40% of female teenage dropouts are mothers. 3. In the United States, the adolescent birth rate is higher among African American and Hispanic teens than among white teens. To some degree, the higher teenage pregnancy rate in these groups reflects the impact of poverty because a disproportionately higher number of African American and Hispanic youths live in poverty. 4. Teens with future goals (i.e., college or job) tend to use birth control more consistently compared with other teens; if they become pregnant, they are also more likely to have abortions.

The nurse is seeing a client who asks about the accuracy of Nageles rule. The nurse explains that accuracy can be compromised under which conditions? Note: Credit will be given only if all correct and no incorrect choices are selected. Select all that apply. 1. There is a history of regular menses every 28 days. 2. Amenorrhea is present and ovulation occurs with breastfeeding. 3. Oral contraception was discontinued, but no regular menstruation was established. 4. There has been 1 or more months of amenorrhea. 5. There is an accurate date for the last menstrual period.

Answer: 2, 3, 4 Explanation: 2. Nageles rule is not always accurate for women who have amenorrhea but are ovulating and conceive while breastfeeding. 3. Nageles rule is not always accurate for women who conceive before regular menstruation is established following discontinuation of oral contraceptives or termination of a pregnancy. 4. Nageles rule is not always accurate for women with markedly irregular periods that include 1 or more months of amenorrhea.

Absolute contraindications to exercise while pregnant include which of the following? Note: Credit will be given only if all correct and no incorrect choices are selected. Select all that apply. 1. Abruptio placentae 2. Placenta previa after 26 weeks gestation 3. Preeclampsia-eclampsia 4. Cervical insufficiency (cerclage) 5. Intrauterine growth restriction (IUGR)

Answer: 2, 3, 4 Explanation: 2. Placenta previa after 26 weeks gestation is an absolute contraindication to exercise. 3. Preeclampsia-eclampsia is an absolute contraindication to exercise. 4. Cervical insufficiency (cerclage) is an absolute contraindication to exercise.

The nurse is preparing a brochure for couples considering pregnancy after the age of 35. Which statements should be included? Note: Credit will be given only if all correct choices and no incorrect choices are selected. Select all that apply. 1. There is a decreased risk of Down syndrome. 2. Preexisting medical conditions can complicate pregnancy. 3. Preterm births are more common. 4. Amniocentesis can be performed to detect genetic anomalies. 5. The increased fertility of women over age 35 makes conception easier.

Answer: 2, 3, 4 Explanation: 2. Preexisting medical conditions, such as hypertension or diabetes, probably play a more significant role than age in maternal well-being and the outcome of pregnancy. 3. The incidence of low-birth-weight infants, preterm births, miscarriage, stillbirth, and perinatal morbidity and mortality is higher among women age 35 or older. 4. Amniocentesis is offered to all women over age 35 to permit the early detection of several chromosomal abnormalities, including Down syndrome; noninvasive analysis of fetal nucleic acid is now commonly recommended to women of advanced maternal age.

A pregnant client complains to the clinic nurse that her varicose veins are causing more discomfort than before. Which recommendations does the nurse make to the client? Note: Credit will be given only if all correct and no incorrect choices are selected. Select all that apply. 1. Increase the time she stands. 2. Walk on a daily basis. 3. Not cross her legs at the knees. 4. Wear support hose. 5. Hyperextend her knee with her feet up.

Answer: 2, 3, 4 Explanation: 2. Regular exercise, such as swimming, cycling, or walking, promotes venous return, which helps prevent varicosities. 3. The client should avoid crossing her legs at the knees because of the pressure it puts on her veins. 4. Supportive hose or elastic stockings may be extremely helpful.

27) A pregnant client at 28 weeks gestation asks the nurse what her baby is like at this stage of pregnancy. How would the nurse describe the fetus? Select all that apply. 1. The fetus is developing subcutaneous fat. 2. The fetus is now opening and closing her eyes. 3. The baby could now breathe on her own, if she were born. 4. The fetus has fingernails and toenails. 5. The fetus is forming surfactant needed for lung function.

Answer: 2, 3, 5 Explanation: 2. At 28 weeks, the eyes begin to open and close. 3. At 28 weeks, the delivered baby can breathe. 5. At 28 weeks, the fetus has the surfactant formed needed for breathing.

11) The nurse in the prenatal clinic will tell the client at 38-weeks gestation to lie on her left side when the client complains of which of the following? Select all that apply. 1. Nausea 2. Pallor 3. Clamminess 4. Constipation 5. Dizziness

Answer: 2, 3, 5 Explanation: 2. Vena caval syndrome can cause pallor, which is relieved when the client turns to lie on her left side. 3. Vena caval syndrome can cause clamminess, which is relieved when the client turns to lie on her left side. 5. Vena caval syndrome can cause dizziness, which is relieved when the client turns to lie on her left side.

What signs would indicate that a pregnant clients urinalysis culture was abnormal? Select all that apply. 1. pH 4.6-8 2. Alkaline urine 3. Cloudy appearance 4. Negative for protein and red blood cells 5. Hemoglobinuria

Answer: 2, 3, 5 Explanation: 2. Alkaline urine could indicate metabolic alkalemia, Proteus infection, or an old specimen. 3. A cloudy appearance could indicate an infection. 5. Hemoglobinuria would be indicated by an abnormal urine color.

19) The clinic nurse is teaching a pregnant client about her iron supplement. Which information is included in the teaching? Note: Credit will be given only if all correct and no incorrect choices are selected. Select all that apply. 1. Iron does not affect the gastrointestinal tract. 2. A stool softener might be needed. 3. Start a low dose, and increase it gradually. 4. Expect the stools to be black and bloody. 5. Iron absorption is poor if taken with meals.

Answer: 2, 3, 5 Explanation: 2. Constipation can be a problem when taking iron, so a stool softener might be needed. 3. To prevent anemia, experts recommend that all pregnant women start on 30 mg/day of iron supplements daily. If anemia is diagnosed, the dosage should be increased to 60 to 120 mg per day of iron. 5. Iron absorption is reduced by 40% to 50% if taken with meals.

The nurse is working with a community group to reduce teen pregnancy rates. Alternatives for the group to consider include which of the following? Note: Credit will be given only if all correct and no incorrect choices are selected. Select all that apply. 1. Avoidance of sex education in schools 2. High-quality reproductive health services 3. Developmentally appropriate sex education 4. Sexuality curriculum based on community beliefs 5. Provision of strategies to enhance life skills

Answer: 2, 3, 5 Explanation: 2. Having access to high-quality reproductive health services is important in reducing adolescent pregnancy rates. 3. Providing developmentally appropriate sex education is an approach that reduces adolescent pregnancy. 5. Programs that provide strategies to enhance life skills are more successful in reducing adolescent pregnancy rates.

24) The nurse is evaluating the goal Client will remain free of opportunistic infections for an HIV-positive pregnant client. The nurse determines the goal was met when the client has which of the following? Note: Credit will be given only if all correct and no incorrect choices are selected. Select all that apply. 1. An absolute CD4+ T-lymphocyte count below 200 2. No complaint of chills or fever during the pregnancy 3. Weight gain of 30 lbs during the pregnancy 4. ESR above 20 mm/hr 5. Normal erythrocyte sedimentation rate maintained during the pregnancy

Answer: 2, 3, 5 Explanation: 2. Not having chills, fever, or a sore throat throughout the pregnancy is an indication the client did not have an infection. 3. Weight gain of 25 to 35 pounds is normal for a pregnancy. This client met the goal for nutrition and remaining infection-free. 5. Having a normal erythrocyte sedimentation rate during the pregnancy is an expected outcome.

When working with pregnant adolescents, the nurse plans on assessing which of the following? Note: Credit will be given only if all correct and no incorrect choices are selected. Select all that apply. 1. High-birth-weight infants 2. Pelvic measurements 3. For sexually transmitted infections 4. Low blood pressure readings 5. Hemoglobin and hematocrit

Answer: 2, 3, 5 Explanation: 2. Teenagers are more likely to have cephalopelvic disproportion. 3. Teenagers are more likely to have sexually transmitted infections. 5. Teenagers are more likely to be anemic, so the hemoglobin and hematocrit must be assessed.

During the initial prenatal visit, the nurse assesses the history of the father of the child for which of the following? Note: Credit will be given only if all correct and no incorrect choices are selected. Select all that apply. 1. Stability of living conditions 2. Blood type and Rh type 3. Significant health problems 4. Nutritional history 5. Current use of tobacco

Answer: 2, 3, 5 Explanation: 2. The father of the fetus should be assessed for blood type and Rh factor. 3. The father of the fetus should be assessed for significant health problems. 5. The father of the fetus should be assessed for current alcohol intake, drug use, and tobacco use.

16) A male client visits the infertility clinic for the results of his comprehensive exam. The exam indicated oligospermia. The client asks the nurse which procedure would assist him and his wife to conceive. The nurses best response would include which of the following? Select all that apply. 1. You might want to consider adoption. 2. An option you might consider is in vitro fertilization. 3. Surrogacy might be your best option. 4. Many couples utilize therapeutic husband insemination. 5. The GIFT procedure has had much success.

Answer: 2, 4 Explanation: 2. The in vitro fertilization procedure is used in cases in which infertility has resulted from male infertility. 4. Therapeutic husband insemination is generally indicated for such seminal deficiencies as oligospermia.

25) A couple is at the clinic for preconceptual counseling. Both parents are 40 years old. The nurse knows that the education session has been successful when the wife makes which statement? Select all that apply. 1. We are at low risk for having a baby with Down syndrome. 2. Our children are more likely to have genetic defects. 3. Children born to parents this age have sex-linked disorders. 4. The tests for genetic defects can be done early in pregnancy. 5. It will be almost impossible for us to conceive a child.

Answer: 2, 4 Explanation: 2. Women 35 or older are at greater risk for having children with chromosome abnormalities. 4. Genetic testing such as amniocentesis and chorionic villus sampling are done in the first trimester.

2) A nurse is discussing the serving sizes in the grains food group with a new prenatal patient in the clinic. Which food equals one serving size from the grains food group? Note: Credit will be given only if all correct choices and no incorrect choices are selected. Select all that apply. 1. One hamburger roll 2. One slice of bread 3. One cup of pasta 4. One tortilla 5. One ounce of dry cereal

Answer: 2, 4, 5 Explanation: 2. One slice of bread is one serving. 4. One tortilla is one serving. 5. One ounce of dry cereal is one serving.

20) A client is being admitted to the labor area with the diagnosis of eclampsia. Which actions by the nurse are appropriate at this time? Note: Credit will be given only if all correct choices and no incorrect choices are selected. Select all that apply. 1. Tape a tongue blade to the head of the bed. 2. Pad the side rails. 3. Have the woman sit up. 4. Provide the client with grief counseling. 5. The airway should be maintained and oxygen administered.

Answer: 2, 5 Explanation: 2. Side rails should be up and padded. 5. Suctioning may be necessary to keep the airway clear.

12) A pregnant client has been admitted with a diagnosis of hyperemesis. Which orders written by the primary healthcare provider are the highest priorities for the nurse to implement? Note: Credit will be given only if all correct choices and no incorrect choices are selected. Select all that apply. 1. Obtain complete blood count. 2. Start intravenous fluid with multivitamins. 3. Check admission weight. 4. Obtain urine for urinalysis. 5. Give a medication to stop the nausea and vomiting.

Answer: 2, 5 Explanation: 2. Starting intravenous fluid with multivitamins is a priority if the client has been vomiting. 5. Giving a medication to stop the nausea and vomiting is a priority.

34) A client tells the nurse that she does not like citrus fruits, and would like suggestions for alternate vitamin C sources. What should the nurse suggest as good sources of vitamin C? Note: Credit will be given only if all correct choices and no incorrect choices are selected. Select all that apply. 1. Barley and brown rice 2. Strawberries and potatoes 3. Buckwheat and lentils 4. Wheat flour and figs 5. Blueberries and broccoli

Answer: 2, 5 Explanation: 2. Strawberries and potatoes are very good sources of vitamin C. 5. Cantaloupes and broccoli are very good sources of vitamin C.

19) A pregnant client who is a lacto-vegetarian asks the nurse for assistance with her diet. What instruction should the nurse give? Note: Credit will be given only if all correct choices and no incorrect choices are selected. Select all that apply. 1. Protein is important; therefore, the addition of one serving of meat a day is necessary. 2. A daily supplement of vitamin B12 is important. 3. The high fiber in a vegetarian diet is dangerous for pregnant women. 4. Eggs are important to add to your diet. Eat six eggs per week. 5. Milk products contain protein, but they are very low in iron.

Answer: 2, 5 Explanation: 2. Supplementation may be recommended for vegans who have difficulty meeting the recommended amounts of vitamin B12 through food sources.

5) What is the increased vascularization causing the softening of the cervix known as? 1. Hegar sign 2. Chadwick sign 3. Goodell sign 4. McDonald sign

Answer: 3 Explanation: 3. Increased vascularization causes the softening of the cervix known as Goodell sign.

24) The client is at 6 weeks gestation, and is spotting. The client had an ectopic pregnancy 1 year ago, so the nurse anticipates that the physician will order which intervention? 1. A urine pregnancy test 2. The client to be seen next week for a full examination 3. An antiserumpregnancy test 4. An ultrasound to be done

Answer: 3 Explanation: 3. A -Subunit radioimmunoassay (RIA) uses an antiserum with specificity for the -subunit of hCG in blood plasma. This test may not only detect pregnancy but also detect an ectopic pregnancy or trophoblastic disease.

4) The nurse is listening to the fetal heart tones of a client at 37 weeks gestation while the client is in a supine position. The client states, Im getting lightheaded and dizzy. What is the nurses best action? 1. Assist the client to sit up. 2. Remind the client that she needs to lie still to hear the baby. 3. Help the client turn onto her left side. 4. Check the clients blood pressure.

Answer: 3 Explanation: 3. During pregnancy the enlarging uterus may put pressure on the vena cava when the woman is supine, resulting in supine hypotensive syndrome. This pressure interferes with returning blood flow and produces a marked decrease in blood pressure with accompanying dizziness, pallor, and clamminess, which can be corrected by having the woman lie on her left side.

8) During her first months of pregnancy, a client tells the nurse, It seems like I have to go to the bathroom every 5 minutes. The nurse explains to the client that this is because of which of the following? 1. The client probably has a urinary tract infection. 2. Bladder capacity increases throughout pregnancy. 3. The growing uterus puts pressure on the bladder. 4. Some women are very sensitive to body function changes.

Answer: 3 Explanation: 3. During the first trimester, the growing uterus puts pressure on the bladder, producing urinary frequency until the second trimester, when the uterus becomes an abdominal organ. Near term, when the presenting part engages in the pelvis, pressure is again exerted on the bladder.

1) A client with a normal prepregnancy weight asks why she has been told to gain 25-35 pounds during her pregnancy while her underweight friend was told to gain more weight. What should the nurse tell the client the recommended weight gain is during pregnancy? 1. 25-35 pounds, regardless of a clients prepregnant weight 2. More than 25-35 pounds for an overweight woman 3. Up to 40 pounds for an underweight woman 4. The same for a normal weight woman as for an overweight woman

Answer: 3 Explanation: 3. Prepregnant weight determines the recommended weight gain during pregnancy. Underweight women are advised to gain 28-40 pounds.

13) The nurse has received a phone call from a multigravida who is 21 weeks pregnant and has not felt fetal movement yet. What is the best action for the nurse to take? 1. Reassure the client that this is a normal finding in multigravidas. 2. Suggest that she should feel for movement with her fingertips. 3. Schedule an appointment for her with her physician for that same day. 4. Tell her gently that her fetus is probably dead.

Answer: 3 Explanation: 3. Quickening, or the mothers perception of fetal movement, occurs about 18 to 20 weeks after the LMP in a primigravida (a woman who is pregnant for the first time) but may occur as early as 16 weeks in a multigravida (a woman who has been pregnant more than once).

22) Whether sensitization is the result of a blood transfusion or maternal-fetal hemorrhage for any reason, what test can be performed to determine the amount of Rh(D) positive blood present in the maternal circulation and to calculate the amount of Rh immune globulin needed? 1. Indirect Coombs test 2. Nonstress test 3. Kleihauer-Betke or rosette test 4. Direct Coombs test

Answer: 3 Explanation: 3. A Kleihauer-Betke or rosette test can be performed to determine the amount of Rh(D) positive blood present in the maternal circulation and to calculate the amount of Rh immune globulin needed.

In early-pregnancy class, the nurse emphasizes the importance of 8-10 glasses of fluid per day. How many of these should be water? 1. 1 to 2 2. 2 to 4 3. 4 to 6 4. 3 to 5

Answer: 3 Explanation: 3. A pregnant woman should consume at least 8 to 12 (8 oz) glasses of fluid each day, of which 4 to 6 glasses should be water.

7) A client calls the urologists office to receive instructions about semen analysis. What does the nurse instruct the client to do? 1. Avoid sexual intercourse 24 hours prior to obtaining a specimen. 2. Use a latex condom to collect the specimen. 3. Expect that a repeat test might be required. 4. Expect a small sample.

Answer: 3 Explanation: 3. A repeat semen analysis might be required to adequately assess the mans fertility potential.

34) A 58-year-old father and a 45-year-old mother gave birth to a baby boy 2 days ago. The nurse assesses a single palmar crease and low-set ears on the newborn. The nurse plans to counsel the couple about which chromosomal abnormality? 1. Trisomy 13 2. Trisomy 18 3. Trisomy 21 4. Trisomy 26

Answer: 3 Explanation: 3. A single palmar crease and low-set ears are characteristics of trisomy 21 (Down syndrome).

The nurse assessing a pregnant African American woman in the first trimester understands that a cultural practice is which of the following? 1. Use of herbs like dandelion during pregnancy to increase lactation 2. Drinking ginseng tea for faintness 3. Eating clay to supply dietary minerals 4. Consulting a spiritual advisor to ensure a healthy pregnancy and birth

Answer: 3 Explanation: 3. African American pregnant women may be guided by their extended family into common practices such as geophagia, the ingestion of dirt or clay, which is believed to alleviate mineral deficiencies.

17) The nurse is assessing a new client in the clinic. The nurse knows that the subjective (presumptive) signs and symptoms of pregnancy include which of the following? 1. Positive urine pregnancy test, enlarged abdomen, and Braxton Hicks contractions 2. Positive urine pregnancy test, amenorrhea, changes in pigmentation of the skin, and softening of the cervix 3. Increase in urination, amenorrhea, fatigue, breast tenderness, and quickening 4. Enlarged abdomen and fetal heartbeat

Answer: 3 Explanation: 3. An increase in urination, amenorrhea, fatigue, breast tenderness, and quickening are all subjective (presumptive) changes of pregnancy.

The nurse is presenting a class of important dos and donts during pregnancy, including travel considerations. What method of travel does the nurse recommend as most appropriate for a client in her 25th week of pregnancy? 1. Automobile 2. Airplane 3. Train 4. None; this client should not travel

Answer: 3 Explanation: 3. As pregnancy progresses, travel by train is generally recommended for long distances.

The nurse is discussing sexual intimacy with a pregnant couple. What should be included in the teaching plan? 1. Intercourse should stop by the beginning of the third trimester. 2. Breast fondling should be discouraged due to the potential for preterm labor. 3. The couple might need to experiment with different positions. 4. Use vaginal lubricant sparingly.

Answer: 3 Explanation: 3. As the uterus enlarges, the couple will have to experiment with different positions.

What would the nurse include as part of a routine physical assessment for a second-trimester primiparous patient whose prenatal care began in the first trimester and is ongoing? 1. Pap smear 2. Hepatitis B screening (HBsAg) 3. Fundal height measurement 4. Complete blood count

Answer: 3 Explanation: 3. At each prenatal visit, the blood pressure, pulse, and weight are assessed, and the size of the fundus is measured. Fundal height should be increasing with each prenatal visit.

19) A woman is experiencing preterm labor. The client asks why she is on betamethasone. Which is the nurses best response? 1. This medication will halt the labor process until the baby is more mature. 2. This medication will relax the smooth muscles in the infants lungs so the baby can breathe. 3. This medication is effective in stimulating lung development in the preterm infant. 4. This medication is an antibiotic that will treat your urinary tract infection, which caused preterm labor.

Answer: 3 Explanation: 3. Betamethasone or dexamethasone is often administered to the woman whose fetus has an immature lung profile to promote fetal lung maturation.

The nurse is teaching an early pregnancy class for clients in the first trimester of pregnancy. Which statement by a client requires immediate intervention by the nurse? 1. When my nausea is bad, I will drink some ginger tea. 2. The fatigue I am experiencing will improve in the second trimester. 3. It is normal for my vaginal discharge to be green. 4. I will urinate less often during the middle of my pregnancy.

Answer: 3 Explanation: 3. Increased whitish vaginal discharge, called leukorrhea, is common in pregnancy. Green discharge is not a normal finding, and indicates a vaginal infection.

7) The nurse is supervising care in the emergency department. Which situation most requires an intervention? 1. Moderate vaginal bleeding at 36 weeks gestation; client has an IV of lactated Ringers solution running at 125 mL/hour 2. Spotting of pinkish-brown discharge at 6 weeks gestation and abdominal cramping; ultrasound scheduled in 1 hour 3. Bright red bleeding with clots at 32 weeks gestation; pulse = 110, blood pressure 90/50, respirations = 20 4. Dark red bleeding at 30 weeks gestation with normal vital signs; client reports an absence of fetal movement

Answer: 3 Explanation: 3. Bleeding in the third trimester is usually a placenta previa or placental abruption. Observe the woman for indications of shock, such as pallor, clammy skin, perspiration, dyspnea, or restlessness. Monitor vital signs, particularly blood pressure and pulse, for evidence of developing shock.

When assisting with a transabdominal sampling, which of the following would the nurse do? 1. Obtain preliminary urinary samples. 2. Have the woman empty her bladder before the test begins. 3. Assist the woman into a supine position on the examining table. 4. Instruct the woman to eat a fat-free meal 2 hours before the scheduled test time.

Answer: 3 Explanation: 3. Clients are placed in a supine position on the table.

The nurse who is counseling a group of middle-school girls on pregnancy avoidance should include which statement? 1. Although sexuality is common in the media, peer pressure to have sex is not an important factor. 2. It has become far less acceptable to give birth during your teenage years than it used to be. 3. Although condom use is growing, there is still an increasing rate of STIs among teens. 4. You have learned enough from your friends and families to understand how pregnancy occurs.

Answer: 3 Explanation: 3. Condom use is increasing, but the rate of STI infections, including HIV, is also rising.

24) Which maternal-child client should the nurse see first? 1. Blood type O, Rh-negative 2. Indirect Coombs test negative 3. Direct Coombs test positive 4. Blood type B, Rh-positive

Answer: 3 Explanation: 3. Direct Coombs test is done on the infants blood to detect antibody-coated Rh-positive RBCs. If the mothers indirect Coombs test is positive and her Rh-positive infant has a positive direct Coombs test, Rh immune globulin is not given; in this case, the infant is carefully monitored for hemolytic disease.

The client has delivered her first child at 37 weeks. The nurse would describe this to the client as what type of delivery? 1. Preterm 2. Postterm 3. Early term 4. Near term

Answer: 3 Explanation: 3. Early term births extend from 37 to 38 weeks gestation.

Which statement, if made by a pregnant client, would indicate that she understands health promotion during pregnancy? 1. I lie down after eating to relieve heartburn. 2. I try to limit my fluid intake to 3 or 4 glasses each day. 3. I elevate my legs while sitting at my desk. 4. I am avoiding exercise to stay well rested.

Answer: 3 Explanation: 3. Elevating the legs can help decrease lower leg edema.

If a woman has the pre-existing condition of diabetes, the nurse knows that she would be prone to what high-risk factor when pregnant? 1. Vasospasm 2. Postpartum hemorrhage 3. Episodes of hypoglycemia and hyperglycemia 4. Cerebrovascular accident (CVA)

Answer: 3 Explanation: 3. Episodes of hypoglycemia and hyperglycemia would be a high-risk factor for a client with pre-existing diabetes.

21) As the couple and their families begin to confront the pain of their loss, many normal manifestations of grief may be present. Which of the following would indicate an emotional response to the loss? 1. Lack of meaning or direction 2. Preoccupation 3. Flat affect 4. Dreams of the deceased

Answer: 3 Explanation: 3. Flat affect would be an emotional response to loss.

24) The nurse is reviewing preconception questionnaires in charts. Which couple are the most likely candidates for preconceptual genetic counseling? 1. Wife is 30 years old, husband is 31 years old 2. Wife and husband are both 29 years old, first baby for husband, wife has a normal 4-year-old 3. Wifes family has a history of hemophilia 4. Single 32-year-old woman is using donor sperm

Answer: 3 Explanation: 3. For families in which the woman is a known or possible carrier of an X-linked disorder, such as hemophilia, the risk of having an affected male fetus is 25%.

2) The nurse has returned from working as a maternal-child nurse volunteer for a nongovernmental organization. After completing a community presentation about this experience, the nurse knows that learning has occurred when a participant states which of the following? 1. Malaria is a chronic disease, and rarely causes fetal loss. 2. Escherichia coli bacteria can cause diarrhea but not stillbirth. 3. Group B streptococci can cause infection and the death of the fetus. 4. Viral infections dont cause fetal death in developing nations.

Answer: 3 Explanation: 3. Group B streptococci can cause ascending infections prior to or after rupture of membranes.

26) If the woman is Rh negative and not sensitized, she is given Rh immune globulin to prevent what? 1. The potential for hemorrhage 2. Hyperhomocysteinemia 3. Antibody formation 4. Tubal pregnancy

Answer: 3 Explanation: 3. If the woman is Rh negative and not sensitized, she is given Rh immune globulin to prevent antibody formation.

A 14-year-old girl is brought to the clinic by her mother. The nurse determines that the teen is about 28 weeks pregnant. The mother states, We knew she was gaining weight, but we cant tell anyone she is pregnant. The nurse understands that the clients mothers behavior exemplifies which of the following? 1. Low self-esteem 2. Anger 3. Shame 4. Ignorance

Answer: 3 Explanation: 3. In families who foster educational and career goals for their children, adolescent pregnancy is often a shock. Anger, shame, and sorrow are common reactions.

Which statement made by the pregnant adolescent would indicate to the nurse that she understood her increased risk of physiologic complications during pregnancy? 1. Its no big deal that I started prenatal care in my seventh month. 2. My anemia and eating mostly fast food are not important. 3. I need to take good care of myself so my baby doesnt come early. 4. Smoking and using crack cocaine wont harm my baby.

Answer: 3 Explanation: 3. In the adolescent age group, prenatal care is the critical factor that most influences pregnancy outcome.

20) A pregnant client confides to the nurse that she is eating laundry starch daily. The nurse should assess the client for which of the following? 1. Alopecia 2. Weight loss 3. Iron deficiency anemia 4. Fecal impaction

Answer: 3 Explanation: 3. Iron deficiency anemia is the most common concern with pica. The ingestion of laundry starch or certain types of clay may contribute to iron deficiency by replacing iron-containing foods from the diet or by interfering with iron absorption.

Which of the following drugs and drug categories can cause multiple fetal central nervous system (CNS), facial, and cardiovascular anomalies? 1. Category C: Zidovudine 2. Category B: Penicillin 3. Category X: Isotretinoin 4. Category A: Vitamin C

Answer: 3 Explanation: 3. Isotretinoin (Accutane), the acne medication, can cause multiple central nervous system (CNS), facial, and cardiovascular anomalies.

The nurse has been asked by a community organization to give a presentation on prevention of teen pregnancy. Which statement indicates appropriate steps toward reduction of the local teen pregnancy rate? 1. Abstinence-only education will be offered in the school and clinics. 2. Classes on how to parent will be mandatory in high school. 3. A low-cost reproductive health clinic will be planned. 4. Parents will be encouraged to avoid discussing sexual activity.

Answer: 3 Explanation: 3. Key strategies for prevention of unintended teen pregnancy and sexual health promotion include the provision of services that ensure accessible and high-quality reproductive health care; sex education programs that provide developmentally appropriate, evidence-based curricula; and youth development strategies to enhance life skills. A low-cost reproductive health clinic would be able to provide these services.

The pregnant 16-year-old is seeing the nurse during a prenatal visit. Based on the clients developmental level, which statement would the nurse expect the client to make? 1. My friends and I all wear totally different styles of clothing. 2. Having a baby will change my college plans. 3. I drink alcohol at parties most weekends. 4. My mom is my best friend.

Answer: 3 Explanation: 3. Middle adolescence (15-17) is a time of experimentation, including drinking alcohol, using other drugs, and sex.

20) Which statement by a pregnant client to the nurse would indicate that the client understood the nurses teaching? 1. Because of their birth relationship, fraternal twins are more similar to each other than if they had been born singly. 2. Identical twins can be the same or different sex. 3. Congenital abnormalities are more prevalent in identical twins. 4. Identical twins occur more frequently than fraternal twins.

Answer: 3 Explanation: 3. Monozygotic twinning is considered a random event and occurs in approximately 3 to 4 per 1000 live births. Congenital anomalies are more prevalent and both twins may have the same malformation.

16) The nurse is creating a poster for pregnant mothers. Which description of fetal development should the nurse include? 1. Four primary germ layers form from the blastocyst. 2. After fertilization, the cells only become larger for several weeks. 3. Most organs are formed by 8 weeks after fertilization. 4. The embryonic stage is from fertilization until 5 months.

Answer: 3 Explanation: 3. Most organs are formed during the embryonic stage, which lasts from the 15th day after fertilization until the end of the 8th week after conception.

17) The community nurse is working with a client at 32 weeks gestation who has been diagnosed with preeclampsia. Which statement by the client would indicate that additional information is needed? 1. I should call the doctor if I develop a headache or blurred vision. 2. Lying on my left side as much as possible is good for the baby. 3. My urine could become darker and smaller in amount each day. 4. Pain in the top of my abdomen is a sign my condition is worsening.

Answer: 3 Explanation: 3. Oliguria is a complication of preeclampsia. Specific gravity of urine readings over 1.040 correlate with oliguria and proteinuria and should be reported to the physician.

11) The nurse is creating a handout on reproduction for teen clients. Which piece of information should the nurse include in this handout? 1. The fertilized ovum is called a gamete. 2. Prior to fertilization, the sperm are zygotes. 3. Ova survive 12-24 hours in the fallopian tube if not fertilized. 4. Sperm survive in the female reproductive tract up to a week.

Answer: 3 Explanation: 3. Ova are considered fertile for about 12 to 24 hours after ovulation.

34) A client has experienced a stillbirth. Which statement by the nurse would be appropriate? 1. You are young. You can try again. 2. At least you have your other children. 3. Im sure you had many dreams and hopes for the future. 4. Its a blessing in disguise.

Answer: 3 Explanation: 3. Perinatal loss is unique in that the parents have not had experiences with the child that was to be, and attachment is based mostly upon hopes and dreams for the future relationship. Page Ref: 945

The prenatal client in her third trimester tells the clinic nurse that she works 8 hours a day as a cashier and stands when at work. What response by the nurse is best? 1. No problem. Your baby will be fine. 2. Do you get regular breaks for eating? 3. Your risk of preterm labor is higher. 4. Standing might increase ankle swelling.

Answer: 3 Explanation: 3. Pregnant women who are employed in jobs that require prolonged standing (more than 3 hours) do have a higher incidence of preterm birth.

9) Which of the following may lead to the development of disseminated intravascular coagulation (DIC), also called consumption coagulopathy, in the mother? 1. Hypertensive disorders 2. Abruptio placentae 3. Prolonged retention of the dead fetus 4. Heritable thrombophilias

Answer: 3 Explanation: 3. Prolonged retention of the dead fetus may lead to the development of disseminated intravascular coagulation (DIC), also called consumption coagulopathy, in the mother.

Screening for gestational diabetes mellitus (GDM) is typically completed between which of the following weeks of gestation? 1. 36 and 40 weeks 2. Before 20 weeks 3. 24 and 28 weeks 4. 30 and 34 weeks

Answer: 3 Explanation: 3. Screening for gestational diabetes mellitus (GDM) is typically completed between 24 and 28 weeks gestation.

35) A client has delivered a stillborn infant at 28 weeks gestation. Which nursing action is appropriate? 1. Discuss funeral options for the baby. 2. Encourage the couple to try to get pregnant again soon. 3. Ask the couple whether or not they would like to hold the baby. 4. Advise the couple that the babys death was probably for the best.

Answer: 3 Explanation: 3. Some parents will hold their infant for a short time before returning him or her to the nurse, whereas others will wish to spend a great deal of time with their infant. Allow the infant to remain with the parents for as long as they desire.

11) During a non-stress test, the nurse notes that the fetal heart rate decelerates about 15 beats during a period of fetal movement. The decelerations occur twice during the test, and last 20 seconds each. The nurse realizes these results will be interpreted as which of the following? 1. A negative test 2. A reactive test 3. A nonreactive test 4. An equivocal test

Answer: 3 Explanation: 3. The FHR acceleration must be at least 15 beats per minute above baseline for at least 15 seconds from baseline to baseline. A nonreactive NST is one that lacks sufficient FHR accelerations over a 40-minute period.

The pregnant client cannot tolerate milk or meat. What would the nurse recommend to the client to assist in meeting protein needs? 1. Wheat bread and pasta 2. Ice cream and peanut butter 3. Eggs and tofu 4. Beans and potatoes

Answer: 3 Explanation: 3. The best food choices that are nondairy and complete proteins alone are eggs and tofu.

4) A woman is hospitalized with severe preeclampsia. The nurse is meal-planning with the client and encourages a diet that is high in what? 1. Sodium 2. Carbohydrates 3. Protein 4. Fruits

Answer: 3 Explanation: 3. The client who experiences preeclampsia is losing protein.

15) A woman asks her nurse what she can do before she begins trying to get pregnant to help her baby, as she is prone to anemia. What would the nurse correctly advise her to do? 1. Get pregnant, then start iron supplementation. 2. Add more carbohydrates to her diet. 3. Begin taking folic acid supplements daily. 4. Have a hemoglobin baseline done now so her progress can be followed.

Answer: 3 Explanation: 3. The common anemias of pregnancy are due either to insufficient hemoglobin production related to nutritional deficiency in iron or folic acid during pregnancy. Folic acid deficiency during pregnancy is prevented by a daily supplement of 0.4 mg (400 micrograms) of folate.

26) The nurse knows that a lecithin/sphingomyelin (L/S) ratio finding of 2:1 on amniotic fluid means which of the following? 1. Fetal lungs are still immature. 2. The fetus has a congenital anomaly. 3. Fetal lungs are mature. 4. The fetus is small for gestational age.

Answer: 3 Explanation: 3. The concentration of lecithin begins to exceed that of sphingomyelin, and at 35 weeks the L/S ratio is 2:1. When at least two times as much lecithin as sphingomyelin is found in the amniotic fluid, RDS is very unlikely.

5) The nurse has completed a presentation on reproduction. Which statement indicates that the teaching has been successful? 1. A male is born with all the sperm he will ever produce. 2. Females create new ova throughout their reproductive life. 3. Ova separate into two unequally sized cells. 4. Each primary spermatocyte divides into four haploid cells.

Answer: 3 Explanation: 3. The first meiotic division produces two cells of unequal size with different amounts of cytoplasm but with the same number of chromosomes.

7) The nurse is teaching a class on perinatal loss to student nurses. What would the nurse explain about the relationship between attachment and the grief response? 1. The mother has no attachment to the fetus before it is born. 2. The severity of the grieving has nothing to do with attachment to the fetus. 3. The intensity of the grief response can be assessed by determining the level of attachment to the anticipated infant. 4. The mother would feel grief only if it were a planned pregnancy.

Answer: 3 Explanation: 3. The intensity to which the grief will be experienced is best understood from the aspect of the level of attachment the grieving person had to the deceased and usually entails finding personal meaning in the loss for successful integration into the grieving persons life. Page Ref: 946

8) Which of the following tests has become a widely accepted method of evaluating fetal status? 1. Contraction stress test (CST) 2. MSAFP test 3. Non-stress test (NST) 4. Nuchal translucency test

Answer: 3 Explanation: 3. The non-stress test (NST) has become a widely accepted method of evaluating fetal status. This test involves using an external electronic fetal monitor to obtain a tracing of the fetal heart rate (FHR) and observation of acceleration of the FHR with fetal movement.

The nurse in the prenatal clinic is seeing a pregnant 16-year-old for the first time. What comment by the young client is the most critical for the nurse to address first? 1. My favorite lunch is burger and fries. 2. Ive been dating my new boyfriend for 2 weeks. 3. On weekends, we go out and drink a few beers. 4. I dropped out of school about 3 months ago.

Answer: 3 Explanation: 3. The nurse responds to this most critical statement because of the danger of fetal alcohol syndrome.

The pregnant client in her second trimester states, I didnt know my breasts would become so large. How do I find a good bra? The best answer for the nurse to make would be which of the following? 1. Avoid cotton fabrics and get an underwire bra; they fit everyone best. 2. Just buy a bra one cup size bigger than usual, and it will fit. 3. Look for wide straps and cups big enough for all of your breast tissue. 4. There isnt much you can do for comfort. Try not wearing a bra at all.

Answer: 3 Explanation: 3. The nurse should instruct the client to get a bra that fits with straps that are wide and do not stretch and a cup that holds all breast tissue comfortably.

The nurse in a prenatal clinic finds that four clients have called with complaints related to their pregnancies. Which call should the nurse return first? 1. Pregnant woman at 7 weeks gestation reporting nasal stuffiness 2. Pregnant woman at 38 weeks gestation experiencing rectal itching and hemorrhoids 3. Pregnant woman at 15 weeks gestation with nausea and vomiting and a 15-pound weight loss 4. Pregnant woman at 32 weeks gestation treating constipation with prune juice

Answer: 3 Explanation: 3. The nurse should return this call first because this patient is the highest priority. A 15-pound weight loss is not an expected finding. Although some nausea is common, the woman who suffers from extreme nausea coupled with vomiting requires further assessment.

A client comes into the prenatal clinic accompanied by her boyfriend. When asked by the nurse why she is there, the client looks down, and the boyfriend states, She says she is pregnant. She constantly complains of feeling tired, and her vomiting is disgusting. What is a priority for the nurse to do at this point? 1. Ask the client what time of the day her fatigue is more common. 2. Recommend that the woman have a pregnancy test done as soon as possible. 3. Continue the interview of the client in private. 4. Give the woman suggestions on ways to decrease the vomiting.

Answer: 3 Explanation: 3. The nurse should suspect that the client is in an abusive relationship. The priority is for the nurse to get the client away from the boyfriend and continue the interview.

8) A client who is 11 weeks pregnant presents to the emergency department with complaints of dizziness, lower abdominal pain, and right shoulder pain. Laboratory tests reveal a beta-hCG at a lower-than-expected level for this gestational age. An adnexal mass is palpable. Ultrasound confirms no intrauterine gestation. The client is crying and asks what is happening. The nurse knows that the most likely diagnosis is an ectopic pregnancy. Which statement should the nurse include? 1. Youre feeling dizzy because the pregnancy is compressing your vena cava. 2. The pain is due to the baby putting pressure on nerves internally. 3. The baby is in the fallopian tube; the tube has ruptured and is causing bleeding. 4. This is a minor problem. The doctor will be right back to explain it to you.

Answer: 3 Explanation: 3. The woman who experiences one-sided lower abdominal pain or diffused lower abdominal pain, vasomotor disturbances such as fainting or dizziness, and referred right shoulder pain from blood irritating the subdiaphragmatic phrenic nerve is experiencing an ectopic pregnancy.

Several adolescent female students are waiting to be seen by the school sexual health clinic nurse. Which student should the nurse see first? 1. 14-year-old whose 17-year-old sister is pregnant 2. 15-year-old who reports using condoms regularly 3. 16-year-old who had chlamydia treated 2 weeks ago 4. 17-year-old with a history of child abuse

Answer: 3 Explanation: 3. This client is the top priority. Teenagers 15 to 19 years old have a high incidence of STIs. The incidence of chlamydial infection is increased in this age group. The presence of such infections during a pregnancy greatly increases the risk to the fetus.

The nurse is presenting a prenatal class to a group of women pregnant for the first time who are all over 35 years of age. The nurse knows that the advantage of waiting until later to start a family is which of the following? 1. That the woman will have an easier labor and delivery. 2. That the baby will be at less of a risk for congenital anomalies. 3. That the woman is more likely to be financially secure. 4. That the woman will be more fertile than a younger woman would.

Answer: 3 Explanation: 3. This delay in starting a family allows women to pursue advanced educational degrees and prepare financially for the impact children will have on their lives.

14) The client in the prenatal clinic tells the nurse that she is sure she is pregnant because she has not had a menstrual cycle for 3 months, and her breasts are getting bigger. What response by the nurse is best? 1. Lack of menses and breast enlargement are presumptive signs of pregnancy. 2. The changes you are describing are definitely indicators that you are pregnant. 3. Lack of menses can be caused by many things. We need to do a pregnancy test. 4. Youre probably not pregnant, but we can check it out if you like.

Answer: 3 Explanation: 3. This is a true statement, and addresses that these changes could be caused by conditions other than pregnancy.

2) A couple is seeking advice regarding what they can do to increase the chances of becoming pregnant. What recommendation can the nurse give to the couple? 1. The couple could use vaginal lubricants during intercourse. 2. The couple should delay having intercourse until the day of ovulation. 3. The woman should refrain from douching. 4. The woman should be on top during intercourse.

Answer: 3 Explanation: 3. This is the correct answer, as douching can alter sperm mobility.

3) The nurse educator is discussing human chromosomes with her students, and knows that the teaching has been effective when a student makes which statement? 1. All humans have 48 chromosomes and 2 sex chromosomes. 2. Human chromosomes are shaped like a Y. 3. Humans have 46 chromosomes, 2 of which are the sex chromosomes. 4. Only certain body cells contain the chromosomes.

Answer: 3 Explanation: 3. This is the correct answer. There are 22 pairs of similar cells and 2 sex chromosomes.

The nurse is talking with the parents of a pregnant 15-year-old. Which statement by one of the parents is expected? 1. Were very happy for her. It will be easier to focus on education with a new baby. 2. Im not going to get involved. She understands how her health insurance works. 3. Her father told her to stop dating that boy. Now look at the trouble shes gotten into. 4. An abortion is the best choice for her. She can deal with our Catholic priest later.

Answer: 3 Explanation: 3. This statement indicates anger, which is to be expected when a parent finds out about a teen daughters pregnancy.

3) The nurse is preparing a client in her second trimester for a three-dimensional ultrasound examination. Which statement indicates that teaching has been effective? 1. If the ultrasound is normal, it means my baby has no abnormalities. 2. The nuchal translucency measurement will diagnose Down syndrome. 3. I might be able to see who the baby looks like with the ultrasound. 4. Measuring the length of my cervix will determine whether I will deliver early.

Answer: 3 Explanation: 3. Three-dimensional ultrasound uses algorithms to vary opacity, transparency, and depth to project an image. This allows curved structures such as the fetal face to be viewed.

During the initial prenatal visit, the pregnant teenager states that she does not know how she got pregnant. The nurse can help to educate her regarding anatomy by doing what? 1. Allowing her to witness a pelvic exam on another teenager 2. Encouraging her to ask her mother about the physiology of pregnancy 3. Including anatomic models and drawings in the teaching session 4. Discussing the process of fetal development with the client

Answer: 3 Explanation: 3. To teach the client about anatomy and physiology, use simple explanations and lots of audiovisuals.

17) The client with an abnormal quadruple screen is scheduled for an ultrasound. Which statement indicates that the client understands the need for this additional antepartal fetal surveillance? 1. After the ultrasound, my partner and I will decide how to decorate the nursery. 2. During the ultrasound we will see which of us the baby looks like most. 3. The ultrasound will show whether there are abnormalities with the babys spine. 4. The blood test wasnt run correctly, and now we need to have the sonogram.

Answer: 3 Explanation: 3. Ultrasound is used to detect neural tube defects. An abnormal serum quadruple screen is not the result of a lab error, and can indicate either an open neural tube defect or trisomy 18 or 21.

14) A diabetic client goes into labor at 36 weeks gestation. Provided that tests for fetal lung maturity are successful, the nurse will anticipate which of the following interventions? Note: Credit will be given only if all correct and no incorrect choices are selected. Select all that apply. 1. Administration of tocolytic therapy 2. Beta-sympathomimetic administration 3. Allowance of labor to progress 4. Hourly blood glucose monitoring 5. Cesarean birth may be indicated if evidence of reassuring fetal status exists

Answer: 3, 4 Explanation: 3. There will be no attempt to stop the labor, as this can compromise the mother and fetus. 4. To reduce incidence of congenital anomalies and other problems in the newborn, the woman should be euglycemic (have normal blood glucose) throughout the pregnancy.

1) A standard ultrasound examination is performed during the second or third trimester and includes an evaluation of which of the following? Note: Credit will be given only if all correct choices and no incorrect choices are selected. Select all that apply. 1. Confirm fetal heart activity. 2. Evaluate the cervix. 3. Determine fetal presentation. 4. Amniotic fluid volume. 5. Fetal number.

Answer: 3, 4, 5 Explanation: 3. A standard ultrasound examination is performed during the second or third trimester and includes an evaluation to determine fetal presentation. 4. A standard ultrasound examination is performed during the second or third trimester and includes an evaluation of amniotic fluid volume. 5. A standard ultrasound examination is performed during the second or third trimester and includes an evaluation of fetal number.

31) The OB-GYN nurse is assessing a pregnant client, and recognizes genetic amniocentesis will be indicated. The nurse makes this conclusion because the indications for genetic amniocentesis include which of the following? Select all that apply. 1. Maternal age under 35 2. Fetus with no abnormalities on ultrasound 3. One child with a chromosome abnormality 4. A family history of neural tube defects 5. Both parents with an abnormal chromosome

Answer: 3, 4, 5 Explanation: 3. Couples who have had a child with trisomy 21, 18, or 13 have approximately a 1% risk or their age-related risk, whichever is higher, of a future child having a chromosome abnormality. 4. Family history of neural tube defects is an indication for genetic amniocentesis. 5. If both parents carry an autosomal recessive disease, they have a 25% chance with each pregnancy that the fetus will be affected.

35) Which couples may benefit from prenatal diagnosis? Select all that apply. 1. Couples including women under the age of 35 2. Couples with an unbalanced translocation 3. Couples with a family history of known or suspected single-gene disorder 4. Couples including women with a teratogenic risk secondary to an exposure or maternal health condition 5. Family history of birth defects and/or intellectual disability

Answer: 3, 4, 5 Explanation: 3. Couples with a family history of known or suspected single-gene disorder (e.g., cystic fibrosis, hemophilia A or B, Duchenne muscular dystrophy) may benefit from prenatal diagnosis. 4. Women with a teratogenic risk secondary to an exposure or maternal health condition (e.g., diabetes, seizure disorder) may benefit from prenatal diagnosis. 5. Family history of birth defects and/or intellectual disability (mental retardation) (e.g., neural tube defects, congenital heart disease, cleft lip and/or palate) may benefit from prenatal diagnosis.

22) A nurse working with couples undergoing genetic testing recognizes which of the following as nursing responsibilities? Select all that apply. 1. Allowing the family to interact with the genetic counselor without interference 2. Giving information about support groups when asked 3. Identifying families at risk for genetic problems 4. Aiding families in coping with the crisis 5. Ensuring continuity of nursing care to the family

Answer: 3, 4, 5 Explanation: 3. The nurse has a responsibility to identify families at risk for genetic problems. 4. The nurse should aid families in coping. 5. The nurse needs to ensure continuity of care to the family.

A pregnant client has a hemoglobin of 10 g/dL and a Hct of 30%. The clinic nurse recognizes the fetus is at risk for which of the following? Note: Credit will be given only if all correct and no incorrect choices are selected. Select all that apply. 1. Macrosomia 2. Respiratory distress syndrome 3. Low birth weight 4. Prematurity 5. Fetal death

Answer: 3, 4, 5 Explanation: 3. Anemia places the fetus at risk for a low birth weight. 4. Anemia places the fetus at risk for premature birth. 5. Anemia places the fetus at risk for fetal death.

The nurse has given a community presentation on adolescent pregnancy. Which statements indicate that further teaching is necessary? Note: Credit will be given only if all correct choices and no incorrect choices are selected. Select all that apply. 1. Low educational achievement is major risk factor for adolescent pregnancy. 2. Teens who are on a low economic trajectory are more likely to become pregnant. 3. When a teen plans to attend college, abortion is less likely. 4. In the United States, the adolescent birth rate is higher among white teens. 5. Teens from low-income households are less likely to become pregnant.

Answer: 3, 4, 5 Explanation: 3. Teens with future goals (i.e., college or job) tend to use birth control more consistently compared with other teens; if they become pregnant, they are also more likely to have abortions. 4. In the United States, the adolescent birth rate is higher among African American and Hispanic teens than among white teens. 5. Poverty is a major risk factor for adolescent pregnancy. As many as 85% of births to unmarried teens occur to those from poor or low-income families.

1) The nurse is explaining the difference between meiosis and mitosis. Which statements would be best? Select all that apply. 1. Meiosis is the division of a cell into two exact copies of the original cell. 2. Mitosis is splitting one cell into two, each with half the chromosomes of the original cell. 3. Meiosis is a type of cell division by which gametes, or the sperm and ova, reproduce. 4. Mitosis occurs in only a few cells of the body. 5. Meiotic division leads to cells that halve the original genetic material.

Answer: 3, 5 Explanation: 3. Meiosis is a special type of cell division by which diploid cells give rise to gametes (sperm and ova). 5. Meiosis creates two cells that contain half the genetic material of the parent cell.

17) The nurse working with a client describes cellular multiplication and how the zygote moves through the fallopian tube, a movement that takes place via what process? 1. A squeezing motion 2. Pushing from another ovum that has not been fertilized 3. Hormone action 4. A very weak fluid current in the fallopian tube resulting from the beating action of ciliated epithelium

Answer: 4 Explanation: 4. This is correct. There is a very weak fluid current in the fallopian tube resulting from the beating action of the ciliated epithelium that lines the tube.

29) The couple at 12 weeks gestation has been told that their fetus has sickle cell disease. Which statement by the couple indicates that they are adequately coping? 1. We knew we were both carriers of sickle cell. We shouldnt have tried to have a baby. 2. If we had been healthier when we conceived, our baby wouldnt have this disease now. 3. Taking vitamins before we got pregnant would have prevented this from happening. 4. The doctor told us there was a 25% chance that our baby would have sickle disease.

Answer: 4 Explanation: 4. A true statement indicates coping. When both parents are carriers of an autosomal recessive disease, there is a 25% risk for each pregnancy that the fetus will be affected.

28) The partner of a client at 16 weeks gestation accompanies her to the clinic. The partner tells the nurse that the baby just doesnt seem real to him, and he is having a hard time relating to his partners fatigue and food aversions. Which statement would be best for the nurse to make? 1. If you would concentrate harder, youd be aware of the reality of this pregnancy. 2. My husband had no problem with this. What was your childhood like? 3. You might need professional psychological counseling. Ask your physician. 4. Many men feel this way. Feeling the baby move in a few weeks will help make it real to you.

Answer: 4 Explanation: 4. Initially, expectant fathers may have ambivalent feelings.The extent of ambivalence depends on many factors, including the fathers relationship with his partner, his previous experience with pregnancy, his age, his economic stability, and whether the pregnancy was planned. The expectant father must first deal with the reality of the pregnancy and then struggle to gain recognition as a parent from his partner, family, friends, coworkers, societyand from his baby as well.

25) A woman calls the clinic and tells a nurse that she thinks she might be pregnant. She wants to use a home pregnancy test before going to the clinic, and asks the nurse how to use it correctly. What information should the nurse give? 1. The false-positive rate of these tests is quite high. 2. If the results are negative, the woman should repeat the test in 2 weeks if she has not started her menstrual period. 3. A negative result merely indicates growing trophoblastic tissue and not necessarily a uterine pregnancy. 4. The client should follow up with a healthcare provider after taking the home pregnancy test.

Answer: 4 Explanation: 4. It is important that clients remember that the tests are not always accurate and they should follow up with a healthcare provider.

35) Which of the following is common in many non-Western cultures and is on the increase in the United States? 1. Ceremonial rituals and rites 2. Cultural assessment 3. Cultural values 4. Cosleeping

Answer: 4 Explanation: 4. Some parents advocate cosleeping or bed sharing (one or both parents sleeping with their baby or young child). Cosleeping, which is common in many non-Western cultures, is on the increase in the United States.

29) The client at 30 weeks gestation with her first child is upset. She tells the prenatal clinic nurse that she is excited to become a mother, and has been thinking about what kind of parent she will be. But her mother has told her that she doesnt want to be a grandmother because she doesnt feel old enough, while her husband has said that the pregnancy doesnt feel real to him yet, and he will become excited when the baby is actually here. What is the most likely explanation for what is happening within this family? 1. Her husband will not attach with this child and will not be a good father. 2. Her mother is rejecting the role of grandparent, and will not help out. 3. The client is not progressing through the developmental tasks of pregnancy. 4. The family members are adjusting to the role change at their own paces.

Answer: 4 Explanation: 4. This is a true statement. With each pregnancy, routines and family dynamics are altered, requiring readjustment and realignment.

8) A client scheduled a laparoscopy. After the procedure, what does the nurse instruct the client to do? 1. Stay on bed rest for 48 hours. 2. Expect to have shoulder and arm pain. 3. Purchase a rectal tube to relieve the gas. 4. Lie on her back to relieve the gas pain after the procedure.

Answer: 4 Explanation: 4. Assuming a supine position may help relieve residual shoulder and chest discomfort caused by any remaining gas.

21) A client has been diagnosed with fallopian tube obstruction and told that her best option for becoming pregnant is with in vitro fertilization. The client asks the nurse about the procedure. What is the nurses best explanation of this procedure? 1. In vitro fertilization (IVF) occurs over a full menstrual cycle. 2. In IVF, a womans ovaries are stimulated by a combination of egg and sperm donations. 3. After ovarian stimulation, you will be inseminated with your partners sperm. 4. The oocytes are aspirated from the ovaries and fertilized in the laboratory.

Answer: 4 Explanation: 4. This is true. The oocytes are aspirated from the clients ovaries and fertilized in the laboratory.

The nurse is caring for a pregnant client. The clients husband has come to the prenatal visit. Which question is the best for the nurse to use to assess the fathers adaptation to the pregnancy? 1. What kind of work do you do? 2. What furniture have you gotten for the baby? 3. How moody has your wife been lately? 4. How are you feeling about becoming a father?

Answer: 4 Explanation: 4. A husbands adaptation to pregnancy includes his feelings about impending fatherhood.

The nurse is providing guidance for a woman in her second trimester of pregnancy and telling her about some of the signs and symptoms that she might experience. Which statement by the client indicates that further teaching is necessary? 1. During the third trimester, I might have frequent urination. 2. During the third trimester, I might have heartburn. 3. During the third trimester, I might have back pain. 4. During the third trimester, I might have a persistent headache.

Answer: 4 Explanation: 4. A persistent headache is not normal or expected. This could be related to the complication of preeclampsia.

7) The nurse is returning phone calls from clients. Which client does the nurse anticipate would not require a serum beta hCG? 1. A client with a risk of ectopic pregnancy 2. A client with spotting during pregnancy 3. A client with previous pelvic inflammatory disease 4. A client with a previous history of twins

Answer: 4 Explanation: 4. A previous history of twins is not a risk factor for ectopic pregnancy. Beta hCG testing is not indicated for this client.

10) The nurse explains to a preconception class that if only a small volume of sperm is discharged into the vagina, an insufficient quantity of enzymes might be released when they encounter the ovum. In that case, pregnancy would probably not result, because of which of the following? 1. Peristalsis of the fallopian tube would decrease, making it difficult for the ovum to enter the uterus. 2. The block to polyspermy (cortical reaction) would not occur. 3. The fertilized ovum would be unable to implant in the uterus. 4. Sperm would be unable to penetrate the zona pellucida of the ovum.

Answer: 4 Explanation: 4. About a thousand acrosomes must rupture to clear enough hyaluronic acid for even a single sperm to penetrate the ovums zona pellucida successfully. If only a small amount of sperm were released, there most likely would be an insufficient quantity of acrosomes to penetrate the zona pellucida of the ovum and allow fertilization.

23) A 15-year-old client has delivered a 22-week stillborn fetus. What does the nurse understand? 1. Grieving a fetal loss manifests with very similar behaviors regardless of the age of the client. 2. Teens tend to withhold emotions and need older adults with the same type of loss to help process the experience. 3. Most teens have had a great deal of contact with death and loss and have an established method of coping. 4. Assisting the client might be difficult because of her mistrust of authority figures.

Answer: 4 Explanation: 4. Adolescents rely heavily on peer support and have a natural mistrust of authority figures, which can make assisting them more difficult

21) Each of the following pregnant women is scheduled for a 14-week antepartal visit. In planning care, the nurse would give priority teaching on amniotic fluid alpha-fetoprotein (AFP) screening to which client? 1. 28-year-old with history of rheumatic heart disease 2. 18-year-old with exposure to HIV 3. 20-year-old with a history of preterm labor 4. 35-year-old with a child with spina bifida

Answer: 4 Explanation: 4. Alpha-fetoprotein (AFP) is a fetal protein that is excreted from the fetal yolk sac during the first 6 weeks of pregnancy. AFP levels can be high or low, with each having different implications for the fetus. If the fetus has a neural tube defect (NTD), the AFP levels will be elevated. NTDs can range from anencephaly to spina bifida. With a past history of a child with spina bifida, this client would be strongly encouraged to have the AFP screening.

The nurse is assessing a newly pregnant client. Which finding does the nurse note as a normal psychosocial adjustment in this clients first trimester? 1. An unlisted telephone number 2. Reluctance to tell the partner of the pregnancy 3. Parental disapproval of the womans partner 4. Ambivalence about the pregnancy

Answer: 4 Explanation: 4. Ambivalence toward a pregnancy is a common psychosocial adjustment in early pregnancy.

26) A newborn has been diagnosed with a disorder that occurs through an autosomal recessive inheritance pattern. The parents ask the nurse, Which of us passed on the gene that caused the disorder? Which answer should the nurse tell them? 1. The female 2. The male 3. Neither 4. Both

Answer: 4 Explanation: 4. An affected individual can have clinically normal parents, but both parents are generally carriers of the abnormal gene.

24) When preparing nutritional instruction, which pregnant client would the nurse consider the highest priority? 1. 40-year-old gravida 2 2. 22-year-old primigravida 3. 35-year-old gravida 4 4. 15-year-old nulligravida

Answer: 4 Explanation: 4. An expectant adolescent must meet the nutritional needs for her own growth in addition to the nutritional needs of pregnancy.

18) The client at 20 weeks gestation has had an ultrasound that revealed a neural tube defect in her fetus. The clients hemoglobin level is 8.5. The nurse should include which statement when discussing these findings with the client? 1. Your low iron intake has caused anemia, which leads to the neural tube defect. 2. You should increase your vitamin C intake to improve your anemia. 3. You are too picky about food. Your poor diet caused your babys defect. 4. You havent had enough folic acid in your diet. You should take a supplement.

Answer: 4 Explanation: 4. An inadequate intake of folic acid has been associated with neural tube defects (NTDs) (e.g., spina bifida, anencephaly, meningomyelocele) in the fetus or newborn.

The nurse is teaching a pregnant 14-year-old client who is at 10 weeks gestation about the expected body changes that will occur during pregnancy. Which client statement indicates that additional information is needed? 1. My breasts are going to get even bigger than theyve gotten over the past couple of years. 2. My belly will gradually get more round, especially from the middle of pregnancy on. 3. I will feel my baby move in about 16 weeks. 4. My diet can stay the same even though Im pregnant.

Answer: 4 Explanation: 4. Because the adolescent is at risk for anemia, she will need education regarding the importance of iron in her diet.

Which third-trimester client would the nurse suspect might be having difficulty with psychological adjustments to her pregnancy? 1. A woman who says, Either a boy or a girl will be fine with me 2. A woman who puts her feet up and listens to some music for 15 minutes when she is feeling too stressed 3. A woman who was a smoker but who has quit at least for the duration of her pregnancy 4. A woman who has not investigated the kind of clothing or feeding methods the baby will need

Answer: 4 Explanation: 4. By the third trimester, the client should be planning and preparing for the baby (for example, living arrangements, clothing, feeding methods).

17) The client with thalassemia intermedia has a hemoglobin level of 9.0. The nurse is preparing an education session for the client. Which statement should the nurse include? 1. You need to increase your intake of meat and other iron-rich foods. 2. Your low hemoglobin could put you into preterm labor. 3. Increasing your vitamin C intake will help your hemoglobin level. 4. You should not take iron supplements.

Answer: 4 Explanation: 4. Folic acid supplements are indicated for women with thalassemia, but iron supplements are not given.

11) A 26-year-old client is 26 weeks pregnant. Her previous births include two large-for-gestational-age babies and one unexplained stillbirth. Which tests would the nurse anticipate as being most definitive in diagnosing gestational diabetes? 1. A 50g, 1-hour glucose screening test 2. A single fasting glucose level 3. A 100g, 1-hour glucose tolerance test 4. A 100g, 3-hour glucose tolerance test

Answer: 4 Explanation: 4. Gestational diabetes is diagnosed if two or more of the following values are met or exceeded after taking the 100 g, 3-hour OGTT: Fasting: 95 mg/dL; 1 hour: 180 mg/dL; 2 hours: 155 mg/dL; 3 hours: 140 mg/dL

34) While doing a prenatal assessment on a woman who has hepatitis B and intends to become pregnant, the nurse explains the impact of the hepatitis B on pregnancy and birth. Which statement does the nurse include in the teaching? 1. Your baby contracted hepatitis B from you when she was conceived. 2. Dont worry about your baby during the birth. Youre more likely to be affected then by the hepatitis B. 3. Your baby will be immune to your hepatitis B. 4. Hepatitis B does not usually affect the course of pregnancy.

Answer: 4 Explanation: 4. Hepatitis B does not usually affect the course of pregnancy.

15) A pregnant woman is having a nipple-stimulated contraction stress test. Which result indicates hyperstimulation? 1. The fetal heart rate decelerates when three contractions occur within a 10-minute period. 2. The fetal heart rate accelerates when contractions last up to 60 seconds. 3. There are more than five fetal movements in a 10-minute period. 4. There are more than three uterine contractions in a 6-minute period.

Answer: 4 Explanation: 4. Hyperstimulation is characterized by contractions that occur more frequently than every 2 minutes or last longer than 90 seconds.

Intercourse is contraindicated if the pregnancy is vulnerable because of which diagnosis? 1. Gestational diabetes 2. Cervical insufficiency (cerclage) 3. Abruptio placentae 4. Placenta previa

Answer: 4 Explanation: 4. Intercourse is contraindicated if the pregnancy is vulnerable because of the diagnosis of threatened spontaneous abortion, placenta previa, or the risk of preterm labor.

The nurse working with mothers over 35 having their first baby knows there are some disadvantages. For what disadvantage would the nurse carefully assess in each client? 1. What kind of insurance the client has for maternity care 2. Whether the client is married 3. Whether the client will continue working after the baby arrives 4. Whether the client has any chronic disease that will have to be addressed

Answer: 4 Explanation: 4. It is important for the nurse to question and assess for any chronic illnesses. The risk of pregnancy complications is higher in women over age 35 who have a chronic condition such as hypertension or diabetes, or who are in poor general health.

4) A woman at 28 weeks gestation is asked to keep a fetal activity record and to bring the results with her to her next clinic visit. One week later, she calls the clinic and anxiously tells the nurse that she has not felt the baby move for more than 30 minutes. Which of the following would be the nurses most appropriate initial comment? 1. You need to come to the clinic right away for further evaluation. 2. Have you been smoking? 3. When did you eat last? 4. Your baby might be asleep.

Answer: 4 Explanation: 4. Lack of fetal activity for 30 minutes typically is insignificant. Movement varies considerably, but most women feel fetal movement at least 10 times in 3 hours.

The nurse is working at a clinic for pregnant teens. What issues related to development will the nurse expect to encounter in most of the pregnant clients? 1. Peer pressure to stop using alcohol once pregnancy is diagnosed 2. Contraception failure that resulted in this pregnancy 3. The father of the babys being emotionally supportive to the client 4. Feelings of not living up to parents expectations

Answer: 4 Explanation: 4. Pregnant teens face risk factors based on the developmental tasks of adolescence. One of these is developing an identity. If the adolescent feels she has not lived up to parental expectations by becoming pregnant, she could adopt a negative identity.

23) The nurse is working with a pregnant 14-year-old. Which statement indicates that additional education is required? 1. Because I am still growing, I need more calories than a pregnant adult. 2. I need to eat fruit and vegetables every day to get enough vitamins. 3. My favorite food is pizza, and I eat it once a week. 4. Because I dont eat breakfast, Ill have to eat more at supper.

Answer: 4 Explanation: 4. Pregnant young adolescents should eat breakfast to ensure the adequate calorie and protein intake needed. In assessing the diet of the pregnant adolescent, the nurse should consider the eating pattern over time, not simply a single days intake.

The clinic nurse is compiling data for a yearly report. Which client would be classified as a primigravida? 1. A client at 18 weeks gestation who had a spontaneous loss at 12 weeks 2. A client at 13 weeks gestation who had an ectopic pregnancy at 8 weeks 3. A client at 14 weeks gestation who has a 3-year-old daughter at home 4. A client at 15 weeks gestation who has never been pregnant before

Answer: 4 Explanation: 4. Primigravida means a woman who is pregnant for the first time.

33) The client has read that the placenta produces hormones that are vital to the function of the fetus. It is evident that that the client understands the function of the placenta when she states that which hormone is primarily responsible for the maintenance of pregnancy past the 11th week? 1. Human chorionic gonadotropin (hCG) 2. Human placental lactogen (hPL) 3. Estrogen 4. Progesterone

Answer: 4 Explanation: 4. Progesterone is a hormone essential for pregnancy. After the 11th week, the placenta produces enough progesterone and estrogen to maintain pregnancy.

23) A pregnant woman is married to an intravenous drug user. She had a negative HIV screening test just after missing her first menstrual period. What would indicate that the client needs to be retested for HIV? 1. Hemoglobin of 11 g/dL and a rapid weight gain 2. Elevated blood pressure and ankle edema 3. Shortness of breath and frequent urination 4. Persistent candidiasis

Answer: 4 Explanation: 4. Signs and symptoms of infections include fever, weight loss, fatigue, persistent candidiasis, diarrhea, cough, and skin lesions (Kaposis sarcoma and hairy leukoplakia in the mouth).

The nurse is providing health teaching to a group of women of childbearing age. One woman states that she is a smoker, and asks about the effect of smoking on her fetus. The nurse tells her that which fetal complication can occur when the mother smokes? 1. Genetic changes in the fetal reproductive system 2. Extensive central nervous system damage 3. Addiction to the nicotine inhaled from the cigarette 4. Low birth rate

Answer: 4 Explanation: 4. Smoking can cause low birth rate.

21) The nurse is assessing a client who is at 35 weeks gestation. What does the nurse expect the client to report at this phase of pregnancy? 1. Nausea and vomiting 2. Maternal ambivalence 3. Emotional shifts from highs to lows 4. Stretch marks on the abdomen

Answer: 4 Explanation: 4. Striae are purplish stretch marks that may develop as the pregnancy progresses.

The nurse developing a care plan for a pregnant teen knows that what plan is best suited to pregnant teens? 1. Assess menstrual history and the presence of any food allergies. 2. Inquire about family relationships and location of the home. 3. Ask what her body image is, then correct her misconceptions. 4. Determine whether there are substance abuse issues.

Answer: 4 Explanation: 4. Substance abuse issues are important to assess during pregnancy of teens.

The nurse working in an adolescent prenatal clinic knows which of the following about the clients who are 15-17 years old? 1. They are more at ease with their individuality. 2. They see authority as resting with parents. 3. They are able to solve problems and make decisions. 4. They seek independence and identify with their peer group.

Answer: 4 Explanation: 4. Teens who are in middle adolescence (15-17 years old) seek independence and identify with their peer group.

7) The client has just been diagnosed as diabetic. The nurse knows teaching was effective when the client makes which statement? 1. Ketones in my urine mean that my body is using the glucose appropriately. 2. I should be urinating frequently and in large amounts to get rid of the extra sugar. 3. My pancreas is making enough insulin, but my body isnt using it correctly. 4. I might be hungry frequently because the sugar isnt getting into the tissues the way it should.

Answer: 4 Explanation: 4. The client who understands the disease process is aware that if the body is not getting the glucose it needs, the message of hunger will be sent to the brain.

The primigravida at 22 weeks gestation has a fundal height palpated slightly below the umbilicus. Which of the following statements would best describe to the client why she needs to be seen by a physician today? 1. Your baby is growing too much and getting too big. 2. Your uterus might have an abnormal shape. 3. The position of your baby cant be felt. 4. Your baby might not be growing enough.

Answer: 4 Explanation: 4. The fundal height at 20-22 weeks should be about even with the umbilicus. At 22 weeks gestation, a fundal height below the umbilicus and the size of the uterus that is inconsistent with length of gestation could indicate fetal demise.

26) A 21-year-old at 12 weeks gestation with her first baby has known cardiac disease, class III, as a result of childhood rheumatic fever. During a prenatal visit, the nurse reviews the signs of cardiac decompensation with her. The nurse will know that the client understands these signs and symptoms if she states that she would notify her doctor if she had which symptom? 1. A pulse rate increase of 10 beats per minute 2. Breast tenderness 3. Mild ankle edema 4. A frequent cough

Answer: 4 Explanation: 4. The hearts signal of its decreased ability to meet the demands of pregnancy includes frequent cough (with or without hemoptysis).

20) A client has just delivered her third child, who was stillborn and had obvious severe defects. Which statement by the nurse is most helpful? 1. Thank goodness you have other children. 2. I am so happy that your other children are healthy. 3. These things happen. They are the will of God. 4. It is all right for you to cry. I will stay here with you.

Answer: 4 Explanation: 4. The nurse needs to let the client know that crying is a normal reaction to the loss event, and that the nurse will stay with her to offer support and understanding.

A client in her third trimester of pregnancy reports frequent leg cramps. What strategy would be most appropriate for the nurse to suggest? 1. Point the toes of the affected leg 2. Increase intake of protein-rich foods 3. Limit activity for several days 4. Flex the foot to stretch the calf

Answer: 4 Explanation: 4. The nurse should advise the client to practice dorsiflexion of feet to stretch affected muscle.

12) The nurse is caring for a couple who are in the labor/delivery room immediately after the delivery of a stillborn baby with visible defects. Which of the following actions by the nurse is appropriate? 1. Discourage the parents from naming the baby. 2. Advise the parents that the babys defects would be too upsetting for them to see. 3. Transport the baby to the morgue as soon as possible. 4. Offer the parents the choice to see and hold the baby.

Answer: 4 Explanation: 4. The nurse should offer the couple the opportunity to see and hold the infant, and reassure the couple that any decision they make for themselves is the right one. Page Ref: 959

13) The nurse is working in an outpatient clinic. Which clients indications most warrant fetal monitoring in the third trimester? 1. Gravida 4, para 3, 39 weeks, with a history of one spontaneous abortion at 8 weeks 2. Gravida 1, para 0, 40 weeks, with a history of endometriosis and a prior appendectomy 3. Gravida 3, para 2, with a history of gestational diabetes controlled by diet 4. Gravida 2, para 1, 36 weeks, with a history of history of preterm labor or cervical insufficiency

Answer: 4 Explanation: 4. The preterm client with a history of preterm labor or cervical insufficiency needs close monitoring for preterm labor onset.

1) During the initial visit with the nurse at the fertility clinic, the client asks what effect cigarette smoking has on the ability to conceive. What is the nurses best response? 1. Smoking has no effect. 2. Only if you smoke more than one pack a day will you experience difficulty. 3. After your first semen analysis, we will determine whether there will be any difficulty. 4. Smoking can affect the quantity of sperm.

Answer: 4 Explanation: 4. The quantity and quality of male sperm are affected by cigarette smoking.

33) A nurse counsels a couple on sex-linked disorders. Both the man and the woman are carriers of the disorder. They ask the nurse how this disorder will affect any children they might have. What is the nurses best response? 1. If you have a daughter, she will not be affected. 2. Your son will be affected because the father has the disorder. 3. There is a 25% chance that your son will have the disorder because the mother has the disorder. 4. There is a 50% chance that your son will be a carrier only.

Answer: 4 Explanation: 4. There is a 50% chance that a carrier mother will pass the normal gene to each of her sons, who will be unaffected.

5) A nurse working in an infertility clinic should include which information in her discussions with the clients? 1. It is important to know the statistics surrounding couples who never learn why they are infertile. 2. Couples should understand the legal controversy concerning therapeutic insemination. 3. Couples should seek marriage counseling before undergoing fertility treatments. 4. Couples should discuss therapeutic insemination and in vitro fertilization as alternatives.

Answer: 4 Explanation: 4. This is the correct answer. This information should be presented to clients so that they are aware of all the alternatives and can make an informed decision.

Recommendations for parents to help their teens avoid pregnancy include which of the following? 1. Parents need to encourage frequent and steady dating. 2. Parents need to let their children set their own goals for the future. 3. Parents need to have their children taught about sex by school programs and community resources. 4. Parents should be clear about their own sexual attitudes and values.

Answer: 4 Explanation: 4. This is true. Parents should be clear about their own sexual attitudes and values in order to communicate clearly with children.

30) The nurse is presenting a class to newly pregnant families. What form of trauma will the nurse describe as the leading cause of fetal and maternal death? 1. Falls 2. Domestic violence 3. Gun accidents 4. Motor vehicle accidents

Answer: 4 Explanation: 4. Trauma from motor vehicle accidents is the leading cause of fetal and maternal death.

21) A clinic nurse is planning when to administer Rh immune globulin (RhoGAM) to an Rh-negative pregnant client. When should the first dose of RhoGAM be administered? 1. After the birth of the infant 2. 1 month postpartum 3. During labor 4. At 28 weeks gestation

Answer: 4 Explanation: 4. When the woman is Rh negative and not sensitized and the father is Rh positive or unknown, Rh immune globulin is given prophylactically at 28 weeks gestation.

28) A 21-year-old woman is at 12 weeks gestation with her first baby. She has cardiac disease, class III, as a result of having had childhood rheumatic fever. Which planned activity would indicate to the nurse that the client needs further teaching? 1. I will be sure to take a rest period every afternoon. 2. I would like to take childbirth education classes in my last trimester. 3. I will have to cancel our trip to Disney World. 4. I am going to start my classes in water aerobics next week.

Answer: 4 Explanation: 4. With the slightest exertion, the clients heart rate will rise, and she will become symptomatic. Therefore, she should not establish a new exercise program.

4) A 20-year-old woman is at 28 weeks gestation. Her prenatal history reveals past drug abuse, and urine screening indicates that she has recently used heroin. The nurse should recognize that the woman is at increased risk for which condition? 1. Erythroblastosis fetalis 2. Diabetes mellitus 3. Abruptio placentae 4. Pregnancy-induced hypertension

Answer: 4 Explanation: 4. Women who use heroin are at risk for poor nutrition, anemia, and pregnancy-induced hypertension (or preeclampsia-eclampsia).

6) The client at 24 weeks gestation is experiencing painless vaginal bleeding after intercourse. The physician has ordered a transvaginal ultrasound examination. Which statements by the client indicate an understanding of why this exam has been requested? Note: Credit will be given only if all correct choices and no incorrect choices are selected. Select all that apply. 1. This ultrasound will show the babys gender. 2. This ultrasound might cause the miscarriage of my baby. 3. This ultrasound carries a risk of creating a uterine infection. 4. This ultrasound can determine the location of my placenta. 5. This ultrasound might detect whether the placenta is detaching prematurely.

Answer: 4, 5 Explanation: 4. Painless bleeding in the second and third trimesters can be a symptom of placenta previa. Transvaginal ultrasound will determine the placental location. 5. Painless bleeding in the second and third trimesters can be a symptom of placenta previa. Transvaginal ultrasound will determine the placental location.

1) The nurse is caring for a client who was just admitted to rule out ectopic pregnancy. Which orders are the most important for the nurse to perform? Note: Credit will be given only if all correct choices and no incorrect choices are selected. Select all that apply. 1. Assess the clients temperature. 2. Document the time of the clients last meal. 3. Obtain urine for urinalysis and culture. 4. Report complaints of dizziness or weakness. 5. Have the lab draw blood for B-hCG level every 48 hours.

Answer: 4, 5 Explanation: 4. Reporting complaints of dizziness and weakness is important, as it can indicate hypovolemia from internal bleeding. 5. Having the lab draw blood for B-hCG levels every 48 hours is important, as the level rises much more slowly in ectopic pregnancy than in normal pregnancy.

32) The mother of a client who has experienced a term stillbirth arrives at the hospital and goes to the nurses desk. The mother asks what she should say to her daughter in this difficult time. What is the nurses best response? Note: Credit will be given only if all correct choices and no incorrect choices are selected. Select all that apply. 1. Use clichs; your daughter will find the repetition comforting. 2. Remind her that she is young and can have more children. 3. Keep talking about other things to keep her mind off the loss. 4. Express your sadness, and sit silently with her if she doesnt respond. 5. Encourage her to talk about the baby whenever she wants to.

Answer: 4, 5 Explanation: 4. Silence is commonly what is needed most, and simply saying Im sorry for your loss might help to facilitate communication. 5. Talking is a way for the client experiencing grief and begin to come to terms with what has happened, and is important for resolution of grief. Intuitive grievers will need to talk about the event.

17) The physician has prescribed the medication clomiphene citrate (Clomid) for a client with infertility. What should the nurses instructions to the woman include? 1. Have intercourse every day of 1 week, starting 5 days after completion of medication. 2. This medication is administered intravenously. 3. Contact the doctor if visual disturbances occur. 4. A contraindication is kidney disease.

Explanation: 3. Side effects of clomiphene citrate include visual symptoms such as spots and flashes.

Chapters 10, 11, and 12

See the cards up next :)

Chapter 16, 17, and 18

See the questions up next :)


Kaugnay na mga set ng pag-aaral

Prep U for Brunner and Suddarth's Textbook of Medical Surgical Nursing, 13th Edition Chapter 15: Oncology: Nursing Management in Cancer Care

View Set

NUR 114 Final Test 1 adaptive quiz

View Set

MATERNAL AND NEWBORN SUCCESS chap 9 High-Risk Intrapartum

View Set

Module 2 - Fair Labor Standards Act

View Set

ATI Activity, ATI Case Studies, PN Pharmacology Online Practice 2017 A&B Questions with Rationales

View Set